Behavioral - Block 2 Practice Questions

Réussis tes devoirs et examens dès maintenant avec Quizwiz!

Children begin to respond to their own names between 7 and 11 months of age. The correct answer is: 7-11 months

At what age child responds to own name? Select one: a. 4-6 months b. 7-11 months c. 16-30 months d. 12-15 months e. 0-3 months

Children begin to use a utensil to feed themselves at about 2 years of age. The correct answer is: 16-30 months

At what age child should be able to feeds self with a spoon? Select one: a. 16-30 months b. 0-3 months c. 7-11 months d. 12-15 months e. 4-6 months

Infants can usually turn over at about 5 months of age. The correct answer is: 4-6 months

At what age child should be able to turns over? Select one: a. 12-15 months b. 0-3 months c. 4-6 months d. 7-11 months e. 16-30 months

The patient is showing evidence of the Klüver-Bucy syndrome, which includes hypersexuality and docility and is associated with damage to the amygdala. The correct answer is: amygdala

A 25-year-old male patient sustains a serious head injury in an automobile accident. He had been aggressive and assaultive, but after the accident, he is placid and cooperative. He also makes in appropriate suggestive comments to the nurses and masturbates a great deal. The area (s) of the brain most likely to be injured in this patient is (are) the Select one: a. reticular system b. left frontal lobe c. hippocampus d. basal ganglia e. right parietal lobe f. amygdala

The amygdala is an important brain area for the evaluation of sensory stimuli with emotional significance. Thus, the brain area most likely to be activated by these photos is the amygdala. The correct answer is: amygdala

A 30-year-old man who has had many negative life experiences becomes upset when he sees photographs of him self taken during these times. The brain area most likely to be activated by these photographs is the Select one: a. nucleus basalis of Meynert b. orbitofrontal cortex c. hypothalamus d. reticular system e. dorsolateral convexity of the frontal lobe f. amygdala

This child is showing learned helplessness, in which an association is made between an aversive stimulus (beatings) and the inability to escape. After the beatings, this child makes no attempt to escape but instead becomes hopeless and apathetic when faced with another beating. Learned helplessness maybe a model system for the development of depression. The correct answer is: learned helplessness

A 4-year-old child who has received beatings in the past, from which he could not escape, appears unresponsive and no longer tries to escape new beatings. This behavior by the child is an example of Select one: a. stimulus generalization b. imprinting c. learned helplessness d. shaping e. modeling

In displacement, the man's personally unacceptable angry feelings toward his wife are taken out on his children. The correct answer is: Displacement

A 40-year-old man who is angry at his ill wife, but does not consciously acknowledge that anger, shouts at his children as soon as he returnshome from work. Which of the following defense mechanisms is described in this scenario? Select one: a. Rationalization b. Sublimation c. Reaction formation d. Regression e. Undoing f. Denial g. Dissociation h. Displacement i. Projection j. Splitting k. Suppression l. Intellectualization

Decreased availability of acetylcholine by blockade of muscarinic acetylcholine receptors (ie, anticholinergic activity) in the CNS is associated with memory problems. Blockade of adrenergic, dopaminergic, histaminergic, and serotonergic receptors are less likely to be associated with memory problems. The correct answer is: Cholinergic

A 55-year-old patient who is taking tiotropium bromide (Spiriva) for chronic obstructive pulmonary disease has memory problems due to the agent's action on which of the following receptors? Select one: a. Cholinergic b. Dopaminergic c. Serotonergic d. Histaminergic e. Adrenergic

The painful blood withdrawal procedure is the unconditioned stimulus. The antiseptic odor in the clinic has become associated with the painful procedure and elicits the same response; it is therefore the conditioned stimulus. The conditioned response, crying in response to the smell of the antiseptic, has been learned. Because crying in response to the pain of an injection is automatic and does not have to be learned, it is the unconditioned response. The correct answer is: unconditioned response → the child's crying when the blood sample is drawn, unconditioned stimulus → the painful blood withdrawal procedure at the child's initial visit, conditioned stimulus → the antiseptic odor that leads to crying on the child's return visit to the laboratory, conditioned response → the child's crying upon the smell of antiseptic

A child comes to the clinical laboratory to have a blood sample drawn for the first time and has a painful experience. The next time the child returns for this procedure, she begins to cry when she smells the odor ofantiseptic in the clinic hallway. Match the following learning theory terms with conditioned/unconditioned stimuli and responses in this scenario: unconditioned response Answer 1Choose...the antiseptic odor that leads to crying on the child's return visit to the laboratorythe painful blood withdrawal procedure at the child's initial visitthe child's crying when the blood sample is drawnthe child's crying upon the smell of antiseptic unconditioned stimulus Answer 2Choose...the antiseptic odor that leads to crying on the child's return visit to the laboratorythe painful blood withdrawal procedure at the child's initial visitthe child's crying when the blood sample is drawnthe child's crying upon the smell of antiseptic conditioned stimulus Answer 3Choose...the antiseptic odor that leads to crying on the child's return visit to the laboratorythe painful blood withdrawal procedure at the child's initial visitthe child's crying when the blood sample is drawnthe child's crying upon the smell of antiseptic conditioned response Answer 4Choose...the antiseptic odor that leads to crying on the child's return visit to the laboratorythe painful blood withdrawal procedure at the child's initial visitthe child's crying when the blood sample is drawnthe child's crying upon the smell of antiseptic

This type of therapy, in which patients with a particular illness (eg, arthritis) meet for communication and practical help, is best described as leaderless group therapy. The correct answer is: leaderless group therapy

Ten arthritis patients meet once per week to talk with each other and to in form each other of new devices and services to help disabled people with everyday tasks. This type of therapy is best described as Select one: a. group therapy b. leaderless group therapy c. brief dynamic psychotherapy d. supportive therapy e. family therapy

Serotonin is metabolized to 5-HIAA. The correct answer is: serotonin

The neurotransmitter metabolized to 5-HIAA (5-hydroxyindoleacetic acid) is Select one: a. norepinephrine b. gamma-aminobutyric acid (GABA) c. acetylcholine (ACh) d. dopamine e. glutamate f. serotonin

The correct answer is: An 80-year-old Anglo-American woman

Which of these patients is most likely to spend the last years of her life in a nursinghome? Select one: a. An 80-year-old Mexican American woman b. An 80-year-old Japanese American woman c. An 80-year-old Anglo-American woman d. An 80-year-old Puerto Rican American woman e. An 80-year-old Vietnamese American woman

This behavior is an example of modeling; the child wants to become like the doctor she admires. In stimulus generalization, a new stimulus that resembles a conditioned stimulus causes a conditioned response. Shaping involves rewarding closer and closer approximations of the wanted behavior until the correct behavior is achieved. Imprinting is the tendency of organisms to make an association with and then follow the first thing they see after birth or hatching. In learned helplessness, an association is made between an aversive stimulus and the inability to escape. The correct answer is: modeling

A 10-year-old child who likes and looks up to her physician states that she wants to become a doctor when she grows up. This behavior by the child is an example of Select one: a. imprinting b. learned helplessness c. modeling d. stimulus generalization e. shaping

The "nuclear family" consists of parents and dependent children (e.g., the boy's sister) living in one household. The great-grandfather, uncle, aunt, and grandmother usually are part of the "extended family." The correct answer is: 10-year-old sister

A 12-year-old child is told to write a report about his 'nuclear family.' To do this task correctly, the report must contain information on his Select one: a. 36-year-old aunt b. 10-year-old sister c. 28-year-old uncle d. 84-year-old great-grandfather e. 55-year-old grandmother

The superego is associated with moral values and conscience, and controls impulses of the id. This teenager who steals from family members and friends and tortures the family cat is showing deficiencies in his superego. Children and adolescents under age 18 years, who have poor superego development, may be diagnosed withconduct disorder. The correct answer is: The superego

A 15-year-old steals from family members and friends. When no one is watching, he also tortures the family cat. Which aspect of the mind is deficient in this teenager? Select one: a. The preconscious mind b. The superego c. The conscious mind d. The ego e. The unconscious mind

This behavior is an example of stimulus generalization. In this example, it occurs when a new conditioned stimulus (the grandmother's white jacket) that resembles the original conditioned stimulus (the nurse's white uniform) results in the conditioned response (crying when he sees his grandmother). The correct answer is: stimulus generalization

A 2-year-old child is afraid of nurses in white uniforms. When his grandmother comes to visit him wearing a white jacket, he begins to cry. This behavior by the child is an example of Select one: a. learned helplessness b. imprinting c. modeling d. stimulus generalization e. shaping

Sedation, increased appetite, and weight gain are side effects of treatment with certain antipsychotic agents. The mechanism most closely associated with these side effects is blockade of histamine receptors since these antipsychotics are not specific for dopamine blockade. Blockade of dopamine receptors by these antipsychotic medications is associated with side effects such as parkinsonism-like symptoms and elevated prolactin levels. The correct answer is: blockade of histamine receptors

A 23-year-old patient shows side effects such as sedation, increased appetite, and weight gain while being treated with antipsychotic medication. Of the following, the mechanism most closely associated with these effects is Select one: a. decreased availability of serotonin b. blockade of serotonin receptors c. blockade of dopamine receptors d. blockade of histamine receptors e. blockade of norepinephrine receptors

Muslim women often prefer to have a female physician, particularly for gynecological or obstetrical problems. In this case, the physician should try to honor the patients wishes. If this is not possible, the patient should be consulted for alternative acceptable strategies, for example, she may suggest having her husband or other family member (e.g., her mother) present when she is examined by the male physician. Trying to impress the patient with one's credentials, or frighten her into adherence are not appropriate or useful strategies. The correct answer is: "I will try to locate a female physician; if I cannot do so, how can I help you be more comfortable with me as your doctor?"

A 24-year-old married Muslim woman, who is experiencing severe pelvic pain, is brought to the emergency room by her husband. When instructed to disrobe and put on a hospital gown, she refuses unless she can be assured that she will be seen by a female physician. The most appropriate statement the male emergency room physician can make at this time is Select one: a. "I cannot help you if you will not cooperate." b. "I will try to locate a female physician; if I cannot do so, how can I help you be more comfortable with me as your doctor?" c. "I am a board-certified physician and am as qualified as a female doctor to examine and treat you." d. "Severe pelvic pain is sometimes life threatening. I must examine you immediately." e. "I will try to locate a female physician but if I cannot do so, I must examine you."

The most appropriate next step in the management of this anxious patient is a medical evaluation, particularly for thyroid function. She has symptoms of an overactive thyroid, including a neck mass (enlarged thyroid gland), exophthalmos and weight loss. People with thyroid hyperactivity may also present with anxiety and insomnia. Psychotherapy, antidepressants, and benzodiazepines can manage the associated symptoms but do not address this patients underlying physical condition. The correct answer is: a medical evaluation

A 25-year-old woman with no psychiatric history has a rapid heart rate and feelings of anxiety which have been present for the past 3 months. The patient has lost 15 pounds and reports that she does not sleep well. Physical examination reveals exophthalmos (bulging eyes) and a neck mass. The most appropriate next step in the management of this patient is Select one: a. an antipsychotic b. a medical evaluation c. a benzodiazepine d. an antidepressant e. psychotherapy

Of the listed factors, their parents'histories of divorce are a risk factor for divorce for this couple. Teenage marriages, short courtship, and differences in socioeconomic and religious backgrounds also put couples at risk for divorce. The correct answer is: Their parents'marital histories

A 26-year-old woman and a 29-year-old man get married after a 2-year engagement. They are both Episcopalian and are both from middle-class families. Both sets of their parents are divorced. Which of the following factors puts this couple at highest risk for divorce? Select one: a. Their parents'marital histories b. Their religious backgrounds c. Their ages d. The length of their engagement e. Their socioeconomic backgrounds

The amobarbital sodium (Amvial) interview is used to determine whether psychological factors are responsible for symptoms in this patient who shows a nonmedically explained loss of sensory function. The correct answer is: amobarbitalsodium interview

A 27-vear-old female patient shows a sudden loss ofmotor function below the waist that cannot be medically explained. To determine whether psychological factors are responsible for this symptom, the most appropriate diagnostic technique is Select one: a. Folstein Mini-Mental State Examination b. CT c. evoked EEG d. Glasgow Coma Scale e. EEG f. amobarbitalsodium interview g. PET

This type of therapy is best described as group therapy, a management technique in which people with a common problem (eg, victims of abuse) get together with a psychotherapist. In leaderless groups, there is no therapist or other person in authority; members of the group provide each other with support and practical help for a shared problem. Brief dynamic psychotherapy is a form of psychoanalytically oriented therapy in which a person works with a therapist to gain insight into the cause of his or her problems. In supportive therapy, a therapist helps a person feel protected and supported during life crises. The correct answer is: group therapy

A 28-year-old woman joins 10 other women who have been abused by their husbands. The women meet weekly and are led by a psychotherapist who is trained in domestic violence issues. This type of therapy is best described as Select one: a. supportive therapy b. group therapy c. family therapy d. leaderless group therapy e. brief dynamic psychotherapy

This 3-year-old girl is showing signs of Rett's disorder that is linked to the X chromosome. Rett's disorder is characterized by loss of social skills after a period of typical functioning as well as hand-wringing and breathing abnormalities. The correct answer is: X

A 3-year-old girl who had been developing typically since birth begins to withdraw socially and then stops speaking altogether. Also, instead of purposeful hand movements, the child has begun to show repetitive hand wringing behavior. The chromosome most likely to be involved in this disorder is chromosome Select one: a. 16 b. 18 c. 21 d. X e. 1

Patients who use a helper animal should be permitted to use the help of the animal in as many situations as possible. Therefore, when it is time for this blind patient to go into the examining room. the physician should allow the patient to be led into the examination room by her guide dog. Seeing-eye dogs are trained to take their masters to many places and are commonly allowed where pet animals are not, for example, public buildings and transportation. There is no reason to believe that the dog will increase the risk of infection for the patient. The correct answer is: allow the patient to be led into the examination room by the dog

A 30-year-old blind patient with a seeing-eve dog comes to a physician for the first time. When it is time for the patient to go into the examining room, the physician should most appropriately Select one: a. tell the patient that the dog will have to stay in the waiting room during the examination for reasons of sanitation b. allow the patient to be led into the examination room by the dog c. take the patient by the arm and lead her into the examining room with the dog following d. ask a member of the office staff to care for the dog during the examination e. make another appointment for the patient when she can come in without the dog

The management technique described here is cognitive/behavioral therapy, a short-term behavioral management technique in which the person is instructed to replace each negative thought with a positive mental image. The correct answer is: Cognitive/behavioral therapy

A 30-year-old depressed man is told to replace each self-deprecating thought with a mental image of victory and praise. Over the next few months, his depression gradually lifts. Which of the following management techniques does this example illustrate? Select one: a. Biofeedback b. Implosion c. Cognitive/behavioral therapy d. Systematic desensitization e. Flooding f. Token economy g. Aversive conditioning

This method of management, systematic desensitization, is based on classical conditioning. The film of people entering elevators in a high-rise building is paired with relaxation. After continued pairing of elevators and relaxation, elevators will no longer induce fear. Later on in treatment, the person will be encouraged to look into a real elevator and finally to ride in one. The correct answer is: classical conditioning

A 30-year-old man who is afraid to ride in an elevator is put into a relaxed state and then shown a film of people entering elevators in a high-rise building. This method of management is based primarily on Select one: a. aversive conditioning b. operant conditioning c. stimulus generalization d. classical conditioning e. reciprocal inhibition

The management technique described here is flooding, a treatment technique for phobias. In flooding, a person is exposed to an overwhelming dose of the feared stimulus or situation - in this case, heights-until he or she is no longer afraid. The correct answer is: Flooding

A 35-year-old man who is afraid of heights is instructed to stand in the observation tower of the Empire State Building and look down from the window until he is no longer afraid. After three visits to the tower each lasting 1 hour, the man is no longer afraid of heights. Which of the following management techniques does this example illustrate? Select one: a. Cognitive/behavioral therapy b. Token economy c. Implosion d. Flooding e. Biofeedback f. Systematic desensitization g. Aversive conditioning

This chronic pain patient is at high risk for depression but at relatively lower risk for drug addiction. Pain patients tend to be undermedicated; it is more likely that this patient is receiving too little rather than too much pain medication. Psychologicaltherapies can be of significant benefit to chronic pain patients. This patient's expression ofpain is related not only to the extent of her pain but also to religious, cultural, and ethnicfactors. The correct answer is: She is at high risk for depression.

A 35-year-old woman with a herniated disc has had back pain for the past 2 years. To help control her pain, she takes an opioid-based medication daily. Which of the following is most likely to be true about this patient? Select one: a. Her expression of pain is related exclusively to the extent of her pain. b. Psychological therapies will not benefit her. c. She is receiving too much pain medication. d. She is at high risk for depression. e. She is at high risk for drug addiction.

This female patient is showing evidence of hvpocortisolism or Addisons disease. This condition is characterized by darkening of the skin, particularly in places not exposed to the sun such as skin creases and inside the mouth. This darkening is not seen in hypercortisolism. pheochromocytoma, or hyper- or hypothyroidism. Hypercortisolism, which also may lead to depression and anxiety is characterized by weight gain, "moon-shaped" face, and skin striae. Depression, dry hair, and weight gain characterize hypothyroidism. while anxiety, fever, weight loss, and elevated heart rate characterize hyperthyroidism. Patients with pheochromocytoma show intense anxiety and elevated VMA in body fluids. The correct answer is: hypocortisolism

A 36-year-old female patient presents to the physician complaining of extreme fatigue and depression. Physical examination reveals a darkening of her skin: particularly in the creases of her hands as well as darkening of the buccal mucosa. The most likely cause of this picture is Select one: a. pheochromocytoma b. hypercortisolism c. hypocortisolism d. hyperthyroidism e. hypothyroidism

The most likely explanation for this difference between the twins (i.e., the one in Japan has and the one in the United States has not been diagnosed with gastric cancer) is that environmental factors are likely to play a role in the development of gastric cancer. If only genetic factors were involved, both women would be likely to have the disease. Adiet high in nitrates such as that eaten in Japan is a risk factor for gastric cancer, but it is not clear that this is the only environmental factor to which the two women are differentially exposed. There is no reason to believe that testing techniques for gastric cancer are different in Japan and in the United States or that exercise can prevent gastric cancer. The correct answer is: environmental factors are likely to play a role in the development of gastric cancer

A 40-year-old Japanese woman born and living in Tokyo has been experiencing gastric bleeding and. after an endoscopy, is diagnosed with gastric cancer. Upon hearing this news, the womans identical twin sister, who has been living in the United States for 20 years and works out at the gym three times per week, has an endoscopy. The results of the sisters test are unremarkable, and no evidence of gastric cancer is found. The most likely explanation for this difference in disease between the twins is that Select one: a. gastric cancer is unrelated to genetic factors b. environmental factors are likely to play a role in the development of gastric cancer c. testing techniques for gastric cancer are better in Japan than in the United States d. a high nitrate diet is probably responsible for gastric cancer e. exercise can prevent gastric cancer

As long as the treatment will not harm the patient, the physician should try to work in conjunction with the healer. Since in this case the folk remedy is innocuous, the patient can continue using it alongwith traditional medical management (e.g., an antihypertensive agent). The physician should not try to separate the patient from his cultural beliefs by refusing to treat him until he stops using the folk healer, questioning the healers training in modern medicine, or doubtingthe value ofthe recommended remedy. It could be dangerous to delay the patients treatment for a month to prove to him that eating corn will not help his condition. The correct answer is: "There are medical treatments for high blood pressure that you can use along with eating corn."

A 40-year-old Mexican American man who has been diagnosed with hypertension tells the physician that a healer, used by many members of his community, told him that eating corn every day will lower his blood pressure. He explains that the healer told him hypertension is a "hot" illness and corn is a "cold" food. If eating corn poses no danger to this patient, what is the doctorsmost appropriate next statement? Select one: a. "Is the healer trained in modern medicine?" b. "I cannot treat you until you stop going to the healer." c. "Try the corn for a month and if your blood pressure is still high. I will give you medication to lower it." d. "There is no medical evidence that corn is helpful for lowering blood pressure." e. "There are medical treatments for high blood pressure that you can use along with eating corn."

Increased TSH, decreased T3 and free T4 as well as tiredness, coarsening of the voice, and osteoporosis indicate that this patient has hypothyroidism. Patients with hypothyroidism not uncommonly present with behavioral symptoms such as those seen in major depressive disorder. Anxiety symptoms such as those seen in panic disorder and OCD are more closely associated with hyperthyroidism. Somatic symptom disorder is diagnosed when physical findings do not adequately explain the patients symptoms. In this patient, the physical findings are significant. The correct answer is: Depression

A 40-year-old woman comes to the doctor with the complaint that she is tired all the time. She also notes that her speaking voice is changing and seems lower than before. Thyroid function tests reveal increased thyroid-stimulating hormone (TSH) and decreased T3 and free T4. Physicalexamination reveals delayed relaxation phase of the muscle strength reflex, and a bone scan reveals evidence of early osteoporosis. This patient is most likely to also show which of the following psychiatric symptoms? Select one: a. Panic attacks b. Hallucinations and delusions c. Somatic symptom disorder d. Obsessions and compulsions e. Depression

Poor appetite, poor sleep, and lack of interest in usual activities characterize patients who have major depression. In this depressed woman, the dexamethasone suppression test is likely to be positive. Apositive result is seen when the synthetic glucocorticoid dexamethasone fails to suppress the secretion of cortisol as it would in a patient with a normal mood. Also, in depression, there may be abnormal growth hormone regulation and melatonin levels, and decreased 5-HIAA. Hypothyroidism maybe associated with depression; hyperthyroidism is more commonly associated with the symptoms of anxiety. The correct answer is: Positive dexamethasone suppression test (DST)

A 40-year-old woman reports that over the past 6 months she has had little appetite, sleeps poorly, and has lost interest in her normal activities. Physical exam is unremarkable. Which of the following is the most likely laboratory finding in this woman? Select one: a. Increased 5-hydroxyindoleacetic acid (5-HIAA) levels b. Normal melatonin levels c. Normal growth hormone regulation d. Positive dexamethasone suppression test (DST) e. Hyperthyroidism

The management technique described here is aversive conditioning, in which a maladaptive but pleasurable stimulus (for this man, sexual interest in children) is paired with painful stimulus (eg, a shock) so that the two become associated. The person now associates sexual interest in children with pain and stops this maladaptive behavior. The correct answer is: Aversive conditioning

A 42-year-old man with sexual interest in children (pedophilia) is given an electric shock each time he is shown a videotape of children. Later, he feels tense around children and avoids them. Which of the following management techniques does this example illustrate? Select one: a. Token economy b. Systematic desensitization c. Cognitive/behavioral therapy d. Biofeedback e. Aversive conditioning f. Flooding g. Implosion

According to the Holmes and Rahe scale, marital separation puts this man at the highest risk for physical illness this year. The other events in this man's life in decreasing order of stressfulness are serious head injury, large mortgage, changing residence, and going on vacation. The correct answer is: Experiencing marital separation

A 45-year-old man has a routine yearly physical examination. While taking the history the doctor determines that during the last year, the patient took out a substantial mortgage and moved to anew house. During the move, he fell and sustained a head injury, which required hospitalization. While recuperating, he and his wife of 10 years went on a 2-week vacation. He recovered completely, but after the vacation, the couple separated. According to the Holmes and Rahe scale, which of these social experiences puts this man at the highest risk for physical illness in the next year? Select one: a. Changing residence b. Going on vacation c. Experiencing marital separation d. Taking out a large mortgage e. Sustaining a serious injury

The doctors most appropriate response to this patient with pain causedby rheumatoid arthritis is to recognize that she is upset because her independence hasbeen lost. Apractical suggestion such as using a long-handled tool is helpful. Telling her towait until her husband comes home or calling a plumber can intensify her feelings of help-lessness. Recommending an arthritis support group maybe useful in the long term but willnot help her with the current problem . Acortisone shot might or might not be help fill butis again not appropriate to deal with the immediate problem. The correct answer is: "People with rheumatoid arthritis often feel that their independence has been lost because of their pain. Perhaps using a tool like a wrench with a long handle will help."

A 45-year-old woman with rheumatoid arthritis calls her physician on a Monday morning because she cannot turn on the bathtub faucet because of the pain in her hands and wrists. She is tearful and says, "My husband has already left for work and my hands hurt too much to turn the water on. Now I can't even take a bath." Which of the following is the doctor's most appropriateresponse? Select one: a. "I know this must be difficult for you, perhaps you can call a plumber." b. "Come to the office right away and I will give you a cortisone shot." c. "Many people with rheumatoid arthritis have difficulty turning faucets. Perhaps you would be interested in joining an arthritis support group." d. "People with rheumatoid arthritis often feel that their independence has been lost because of their pain. Perhaps using a tool like a wrench with a long handle will help." e. "I sympathize with you. Unfortunately, it looks like your only option is to wait until your husband comes home."

The management technique described here is systematic desensitization. In this example, the child made an erroneous negative association between dogs and pain when she was injured in the presence of the dog. In systematic desensitization, increasing doses of the frighten ing stimulus (eg, dogs) are paired with a relaxing stimulus (eg, the favorite CD) to provoke a relaxation response in situations involving the frightening stimulus. Later in treatment, this child will remain in a relaxed state when she is exposed to a living dog. The correct answer is: Systematic desensitization

A 5-year-old child, who at age 2 years was playing with a large dog when a ceiling tile fell on her head, is now so afraid of dogs that she refuses to go to the park because dogs are there. Medical examination is unremarkable, and the child's motor, social, and cognitive development are typical for her age. To manage the child's fear of dogs, the physician first recommends that her father carry a small toy dog very gradually toward her while she is listening to her favorite CD. Which of the following psychological therapies does this example illustrate? Select one: a. Flooding b. Token economy c. Implosion d. Cognitive/behavioral therapy e. Systematic desensitization f. Biofeedback g. Aversive conditioning

The technique described here (ie, biofeedback) is based primarily on operant conditioning. Reciprocal inhibition is the mechanism that prevents one from feeling two opposing emotions at the same time (eg, relaxation and fear) and is associated with systematic desensitization. In aversive conditioning, classical conditioning is used to pair a maladaptive but pleasurable stimulus with an aversive or painful stimulus so that the two become associated and the person stops engaging in the maladaptive behavior. The correct answer is: operant conditioning

A 50-year-old male hypertensive patient is given ongoing blood pressure readings as he uses mental relaxation techniques to try to lower his blood pressure. After four sessions using this technique, his blood pressure is lower. This method of blood pressure reduction is based primarily on Select one: a. operant conditioning b. stimulus generalization c. classical conditioning d. reciprocal inhibition e. aversive conditioning

This womans symptoms (e.g., anxiety, fever, weight loss, and flushing) indicate that she has hyperthyroidism. People commonly describe their perception of a rapid heartbeat as "palpitations." The correct answer is: Hyperthyroidism

A 50-year-old woman without a previous psychiatric history reports that over the past few months she has begun to experience intense anxiety and has lost 15 pounds. The patient also complains of "flushing and palpitations." Which of the following is the most likely laboratory finding in this woman? Select one: a. Normal melatonin levels b. Hyperthyroidism c. Normal growth hormone regulation d. Positive DST e. Increased 5-HIAA levels

This patient with abdominal pain, the false belief that his wife is trying to poison him (a delusion), and urine with a purple-red color is most likely to have acute intermittent porphyria, a disorder that is associated with psychiatric symptoms such as delusions. Acute intermittent porphyria is a metabolic disorder in which toxic porphyrins accumulate in tissue leading to high levels of porphobilinogen in urine which colors it purple-red. Purple-red urine is not seen in the serotonin syndrome (high 5-HIAA), Cushings disease (high cortisol), pheochromocytoma (high VMA), or diabetes (high glucose). The correct answer is: porphobilinogen

A 55-year-old male patient with no history of psychiatric illness is admitted to the hospital complaining of intense abdominal pain. He states that over the past few days his wife has been giving him food that is poisoned so that she can kill him and be with another man. The wife states that she loves her husband and would never harm him or leave him. When the patients urine is collected, it appears purplish red in color. Urine testing is most likely to reveal an elevated level of Select one: a. cortisol b. vanillylmandelic acid (VMA) c. porphobilinogen d. 5-hydroxyindoleacetic acid (5-HIAA) e. glucose

The Bender Visual Motor Gestalt Test is used to evaluate visual and motor ability by reproduction of designs. The Luria-Nebraska neuropsychological battery is used to determine cerebral dominance and to identify specific types of brain dysfunction, while the Halstead-Reitan battery is used to detect and localize brain lesions and determine their effects. The dexamethasone suppression test is used to predict which depressed patients will respond well to treatment with antidepressant agents or electroconvulsive therapy. The electroencephalogram (EEG), which measures electrical activity in the cortex, is useful in diagnosing epilepsy and in differentiating delirium from dementia. The correct answer is: Bender Visual Motor Gestalt Test

A 57-vear-old male patient who has had a stroke cannot copy a design drawn by the examiner. The test that the examiner is most likely to be using to evaluate this patient is the Select one: a. electroencephalogram (EEG) b. dexamethasone suppression test (DST) c. Luria-Nebraska neuropsychologicalbattery d. Bender Visual Motor Gestalt Test e. Halstead-Reitan battery

To reduce this patients likelihood ofpsychological problems in the intensive care unit (ICU), the physician should explain the need for and function of the mechanical ventilator and any other mechanical support that he will need. The physician should also encourage visits from family and communication between patient and staff. The patient should also be encouraged to control aspects of his environment (e.g., lighting level). Outside stimuli (e.g., light) should be increased rather than decreased, for example, placing the patients bed near a window. The correct answer is: explain the need for and function of the mechanical ventilator

A 65-vear-old male patient is scheduled for cardiac surgery. After the surgery he will be in the intensive care unit (ICU) for about 24 hours and will require a mechanical ventilator. To reduce this patient's likelihood of psychological problems in the ICU, the physician should Select one: a. limit visits from his family b. reduce his exposure to ambient light c. explain the need for and function of the mechanical ventilator d. have the nurses control the patient's lighting level e. discourage communication between the patient and the staff

Dominance for language in both right-handed and le ft-handed people is usually in the le ft hemisphere of the brain. Perception, musical ability, artistic ability, and spatial relations primarily are functions of the right side of the brain. The correct answer is: Language

A 65-year-old fem ale patient has had a stroke affecting the left hemisphere of her brain. Which of the following functions is most likely to be affected by the stroke? Select one: a. Language b. Perception c. Artistic ability d. Spatial relations e. Musical ability

The most appropriate next step in the management of this patient, who has no previous history of psychiatric illness, is to schedule a medical evaluation. This patient has symptoms of depression including sleep problems, inappropriate guilt, suicidal ideation, and significant weight loss. Depression is not typically seen in normal aging. Because pancreatic cancer and other gastrointestinal cancers as well as diabetes and hypothyroidism not uncommonly present with depression in the elderly, this patient should be evaluated for such conditions prior to treating her depression. Psychotherapy, antidepressants, and antipsychotics do not address this patients underlying physical problem. The correct answer is: to schedule a medical evaluation

A 68-year-old physician with no history of psychiatric problems reports that over the past 3 months she has been having difficulty sleeping through the night and has lost her appetite for food. She states that if she had been a better doctor, some of her patients would not have died, and she expresses strong negative feelings about not spending more time with her children when they were young. The patient also reports that she has lost 25 pounds since the previous year without dieting. The most appropriate next step in the management of this patient is Select one: a. to reassure her that what she is feeling is part of the typical aging process b. an antidepressant c. psychotherapy d. to schedule a medical evaluation e. an antipsychotic

This Latino patient who reports that she communicates with her dead husband is probably not experiencing a delusion (i.e., a false belief not shared by others). Rather, she is most likely to be reporting a cultural phenomenon based on the belief, in some Latino cultures, that the line between the dead and the living is blurred. As further evidence that this is not a delusion the patient shows no evidence of a thought disorder. Thus, she does not need to take an antipsychotic such as risperidone. There is no evidence that she either believes her husband is alive or that she is disturbed by these experiences. The correct answer is: "Do other people in the Latino community believe that the living and the dead communicate with each other?"

A 70-year-old Latino woman, whose husband died 4 months ago, calmly tells her physician that she and her husband still communicate with each other. The patient shows no evidence of a thoughtdisorder, and her physical examination is unremarkable. Which of the following is the most appropriate question or statement from the physician at this time? Select one: a. "How do you feel when your husband communicates with you?" b. "I would like you to take a medication called risperidone for the next few months." c. "Most people do not think that they can communicate with the dead." d. "Do you believe that your husband is still alive?" e. "Do other people in the Latino community believe that the living and the dead communicate with each other?"

This type of therapy, in which a child with a behavior problem and his family meet with a therapist, is best described as family therapy. Family therapy is based on the idea that psychopathology in one family member (e. G., a child) reflects dysfunction of the entire fam ily system. The correct answer is: family therapy

A 9-year-old boy who is angry and resentful toward adults (oppositional defiant disorder) meets with a therapist for 2 hours each week, along with his parents and his sister. After 6 months, the boy's oppositional behavior toward adults has improved. This type of therapy is best described as Select one: a. brief dynamic psychotherapy b. family therapy c. leaderless group therapy d. group therapy e. supportive therapy

Scores below 18 on the Folstein Mini-Mental State Examination indicate significant cognitive impairment. This test does not evaluate calculating ability or intelligence. Although the patient is impaired, it is not clear what caused the problem or whether she needs to be placed in an assisted living facility. The correct answer is: is cognitively impaired

A college-educated 72-year-old female patient has scored 15 on the Folstein Mini-Mental State Examination. From this score, the physician can conclude that this patient probably Select one: a. is cognitively impaired b. has lower than normal intelligence c. is showing typical behavior d. should be placed in an assisted living facility e. cannot calculate simple sums

Young immigrant men, such as the 28-year-old uncle, are at higher risk for psychiatric symptoms when entering a new culture than are any other gender or age group. This is because they lose the most status on leaving their old culture and because, unlike other groups that can stay at home among their families, young men often must get out into the new culture to work and make a living. The correct answer is: 28-year-old uncle

A large extended family immigrates to the United States. The person in the family who is at highest risk for psychiatric symptoms after the move is the Select one: a. 84-year-old great-grandfather b. 55-year-old grandmother c. 28-year-old uncle d. 10-year-old sister e. 36-year-old aunt

The management technique described here is implosion, a management technique related to flooding in which the person is instructed to imagine extensive exposure to a feared stimulus (driving a car) until he or she is no longer afraid. The correct answer is: Implosion

A man who is afraid to drive is told to imagine driving a car from the northernmost border to the southernmost border of the State of New Jersey. Eventually, he is able to drive without fear. Which of the following management techniques does this example illustrate? Select one: a. Implosion b. Token economy c. Flooding d. Aversive conditioning e. Cognitive/behavioral therapy f. Biofeedback g. Systematic desensitization

Intravenous administration of sodium lactate can help identify individuals with panic disorder since it can provoke a panic attack in such patients. The correct answer is: panic disorder

A phvsician administers sodium lactate intravenously to a 28-year-old woman. Using this technique, the physician is trying to provoke, and thus confirm, the patients diagnosis of Select one: a. panic disorder b. delirium c. major depressive disorder d. conversion disorder e. malingering

Statistically, a middle-aged African American patient has a lower likelihood of suicide than a White American patient ofthe same age. However, when compared to White American patients. African American patients have a higher likelihood of stroke, asthma, hypertension, and prostate cancer as well as heart disease, tuberculosis, diabetes, and AIDS. The correct answer is: suicide

A physician has two 56-year-old male patients. One of them is African American and one is White American. Statistically, the African American patient has a lower likelihood of Select one: a. prostate cancer b. hypertension c. stroke d. asthma e. suicide

Memory of the details of the Krebs cycle, while no longer in the forefront of the medical student's mind, can be recalled relatively easily 1 week after the examination. This memory therefore resides in the preconscious mind. The unconscious mind contains repressed thoughts and feelings, which are not available to the conscious mind. The conscious mind contains thoughts that a person is currently aware of. The id contains instinctive sexual and aggressive drives and is not influenced by external reality. The ego also controls the expression of the id, sustains satisfying interpersonal relationships, and. through reality testing, maintains a sense of reality about the body and the external world. The correct answer is: preconscious mind

About 1 week after her final examination for a biochemistry course, a medical student's knowledge of the details of the Krebs cycle is most likely to reside in her Select one: a. conscious mind b. unconscious mind c. ego d. superego e. preconscious mind

Prenatal vitamins contain iron and this child has taken an iron overdose.The physicians next step in management of an iron overdose is to give deferoxaminemesylate, d-Penicillamine, ethylenediaminetetraacetic acid (EDTA), bisphosphonate. andflumazenilare used to manage overdoses of copper, lead, vitamin D. and benzodiazepinesrespectively. The correct answer is: deferoxamine mesylate

After a 5-year-old child takes his mother's prenatal vitamins, he throws up material that looks like coffee grounds and then loses consciousness. In the emergency room, the physician's next step in pharmacological management is to give this child Select one: a. ethylenediaminetetraacetic acid (EDTA) b. flumazenil c. deferoxamine mesylate d. d-penicillamine e. bisphosphonate

Because of the disorienting nature of the ICU, delirium is commonly seen in ICU patients. Panic disorder, obsessive-compulsive disorder, illness anxiety disorder, and somatization disorder are no more common in ICU patients than in the general population. The correct answer is: Delirium

During stay in the ICU alter surgery, a patient is most likely to experience which of the following disorders? Select one: a. Somatic symptom disorder b. Panic disorder c. Illness anxiety disorder d. Delirium e. Obsessive-compulsive disorder

The management technique described here is token economy. In token economy, the desired behavior (eg, grooming) is reinforced by a token (eg, a coupon that can be exchanged for dessert) and the person increases her behavior to gain the reward (eg, dessert). In systematic desensitization, increasing doses of a frightening stimulus are paired with a relaxing stimulus to provoke a relaxation response in situations involving the frightening stimulus. Flooding is a management technique for phobias in which a person is exposed to an overwhelming dose of the feared stimulus or situation until he or she is no longer fearful. In implosion, a person is exposed to an imagined, rather than actual, overwhelming dose of a feared stimulus or situation. In biofeedback, a person is given ongoing physiologic in formation, which acts as reinforcement. In aversive conditioning, a maladaptive but pleasurable stimulus is paired with a painful stimulus so that the two become associated and the maladaptive behavior disappears. In cognitive/behavioral therapy, a person is helped to identify distorted, negative thoughts and to replace them with positive, self-assuring thoughts. The correct answer is: Token economy

Each time she combs her hair, a 20-year-old woman with Level 3 autism spectrum disorder receives a coupon that can be exchanged for dessert in the cafeteria. Her grooming behavior subsequently improves. Which of the following psychological management techniques does this example illustrate? Select one: a. Systematic desensitization b. Token economy c. Implosion d. Flooding e. Aversive conditioning f. Biofeedback g. Cognitive/behavioral therapy

Agranulocytosis is seen particularly in patients taking clozapine, an antipsychotic, or carbamazepine. an anticonvulsant that is used to treat bipolar disorder. Lithium, amobarbital sodium, dexamethasone. and sodium lactate are not specifically associated with agranulocytosis. The correct answer is: Clozapine

Four weeks after he begins to take a new medication, a 28-year-old male psychiatric patient develops fever and sore throat. He reports feeling tired, and blood studies reveal a white blood cell (WBC) count of less than 2,000. This patient is most likely to be taking which of the following agents? Select one: a. Sodium lactate b. Lithium c. Amobarbital sodium d. Clozapine e. Dexamethasone

Since the placebo response is based in part on activation of the endogenous opioid system, it will be blocked by naloxone, and this patient's pain will be unchanged. This experiment will not necessarily affect her response to opioids in the future. The correct answer is: be unchanged

In a clinical experiment, a 48-year-old female patient with chronic pain who, in the past, has responded to placebos is given naloxone. Shortly thereafter the patient is given an inert substance that she believes is a painkiller. After the patient receives the inert substance, her pain is most likely to Select one: a. increase b. be unchanged c. decrease d. respond to lower doses of opioids than previously e. fail to respond to opioids in the future

In the United States, no physician-to-patient transmission of HIVhas been confirmed. The correct answer is: fewer than 50

In the United States, the number of patients confirmed to have contracted HIV from their physicians is Select one: a. between 101 and 200 b. fewer than 50 c. more than 300 d. between 51 and 100 e. between 201 and 300

In crease in penis width, development of the glans, and darkening of scrotal skin characterize Tanner stage 4. Stage 1 is characterized by slight elevation of the papillae and stage 2 by the presence of scant, straight pubic hair, testes enlargement, development of texture in scrotal skin, and slight elevation of breast tissue. In stage 3, pubic hair in creases over the pubis and becomes curly, and the penis in creases in length; in stage 5, male and fem ale genitalia are much like those of adults. The correct answer is: 4

Increase in penis width, development of the glans, and darkening of scrotal skin characterize Tanner stage: Select one: a. 4 b. 5 c. 2 d. 1 e. 3

Sublimation, expressing an unacceptable emotion in a socially acceptable way. is classified as a mature defense mechanism. Denial, dissociation, regression, and intellectualization are all classified as less mature defense mechanisms. The correct answer is: Sublimation

Of the following defense mechanisms, which is considered the most mature? Select one: a. Intellectualization b. Sublimation c. Denial d. Regression e. Dissociation

Psychological stress engendered by this patients pain is likely to result in increased release of adrenocorticotropic hormone (ACTH) and cortisol. This, in turn, results in decreased function of the immune system as reflected in decreased lymphocyte response to mitogens and function of natural killer cells. The correct answer is: release of adrenocorticotropic hormone (ACTH)

Psychological stress is most likely to result in increased Select one: a. function of the immune system b. release of adrenocorticotropic hormone (ACTH) c. cortisol suppression d. lymphocyte response to mitogens e. function of natural killer cells

Remembering that school closes early before Thanksgiving Day every year is an example of semantic memory. Semantic memory is a type of declarative memory that involves remembering general knowledge about the world. Episodic memory involves remembering personally experienced events, procedural memory involves remembering things one does automatically, and working m emory involves remembering recent in formation. The correct answer is: Semantic

Remembering that school closes early before Thanksgiving Day every year is an example of which of the following types of memory? Select one: a. Episodic b. Working c. Procedural d. Semantic

Dopamine hypoactivity in the mesocortical tract is associated with the negative symptoms of schizophrenia (and see Chapter 11). Dopamine hyperactivity in the mesolimbic tract is associated with the positive symptoms of schizophrenia. Dopamine acts on the tuberoinfundibular tract to inhibit the secretion of prolactin from the anterior pituitary. The cerulocortical tract is associated with the action of norepinephrine, while the raphe cortical tract is associated with the action of serotonin. The correct answer is: mesocortical tract

The brain pathway most closely associated with the display of negative symptoms in schizophrenia is the Select one: a. tuberoinfundibular tract b. cerulocortical tract c. raphe cortical tract d. mesolimbic tract e. mesocortical tract

When the management regimen is complex and the patient does not speak English, the physician's best choice is to call in a professional translator so that he can explain the instructions directly to his patient (in this case, the elderly woman). Communicating as directly as possible with the patient is particularly important in cultures in which adult children may protect an elderly relative from a negative medical diagnosis (e.g., Asian and Hispanic cultures). Thus, in translating the information, or monitoring the treatment, thedaughter may not relay the complete picture to the elderly patient. Writing the instructionsdown in English to be translated later is not appropriate because it is uncertain how andwhen the translation will be done. Since the doctor can call in a translator, there is no reasonto refer the patient to another doctor. In any case, referrals should be made only for medicalreasons. The correct answer is: call in a professional translator to explain the instructions to the patient

The daughter of a 65-year-old Vietnamese woman brings her mother in for management of a serious medical condition. The older woman, who lives with her daughter, is alert and oriented. The management regimen is quite complex, and the older woman does not speak English. To best convey the needed information to this patient, the physician should Select one: a. explain the instructions to the daughter and have her monitor the patients treatment b. write the instructions down in English to be translated for the patient later c. call in a professional translator to explain the instructions to the patient d. ask the daughter to translate the instructions to the patient e. refer the patient to a doctor who speaks Vietnamese

While acetylcholine (ACh) is the major neurotransmitter implicated in Alzheimer's disease, abnormalities in glutamate are seen in both Alzheimer's disease and schizophrenia. The correct answer is: glutamate

The major neurotransmitter implicated in both Alzheimer's disease and schizophrenia is Select one: a. norepinephrine b. dopamine c. gamma-aminobutyric acid (GABA) d. serotonin e. glutamate f. acetylcholine (ACh)

Blockade of serotonin reuptake by presynaptic neurons is the primary action of the antidepressant fluoxetine. The correct answer is: serotonin

The major neurotransmitter involved in the antidepressant action of fluoxetine (Prozac) is Select one: a. acetylcholine (ACh) b. dopamine c. serotonin d. norepinephrine e. gamma-aminobutyric acid (GABA) f. glutamate

The major reason that patients who could benefit from psychoanalytically oriented psychotherapy do not receive it is that they often believe it is expensive and time consuming. Less commonly, people do not want to reveal their histories and personal problems to strangers, are not interested in exploring their childhoods, or feel uncomfortable in the therapeutic setting. The correct answer is: believe that it is expensive and time consuming

The major reason that patients who could benefit from psychoanalytically oriented psychotherapy do not receive it is that they often Select one: a. believe that it is expensive and time consuming b. have little interest in exploring their childhoods c. do not want to reveal their histories to strangers d. do not feel comfortable in the therapeutic setting e. do not want to reveal their personal problems to strangers

The auditory evoked EEG can be used to assess whether this child can hear. Evoked EEGs measure electrical activity in the cortex in response to sensory stimulation. The correct answer is: evoked EEG

To determine whether a 3-month-old infant is able to hear sounds, the most appropriate diagnostic technique is Select one: a. amobarbitalsodium interview b. evoked EEG c. Folstein Mini-Mental State Examination d. Glasgow Coma Scale e. PET f. EEG g. CT

Positron emission tomography (PET) localizes physiologically active brain areas by measuring glucose metabolism. Thus, this test can be used to determine which brain area is being used during a specific task (e.g., translating a passage written in Spanish). The correct answer is: positron emission tomography (PET)

To determine which brain area is physiologically active when a 44-year-old male patient is translating a paragraph from Spanish to English, the most appropriate diagnostic technique is Select one: a. electroencephalogram (EEG) b. evoked EEG c. positron emission tomography (PET) d. amobarbital sodium (Amytal)interview e. Folstein Mini-Mental State Examination f. Glasgow Coma Scale g. computed tomography (CT)

Electroencephalogram (EEG) measures electrical activity in the cortex and can be useful in differentiating delirium (abnormal EEG) from dementia (usually normal EEG). The correct answer is: EEG

To differentiate delirium from dementia in a 75-year-old male patient, the most appropriate diagnostic technique is Select one: a. PET b. evoked EEG c. amobarbital sodium interview d. CT e. EEG f. Folstein Mini-Mental State Examination g. Glasgow Coma Scale

Computed tomography (CT) identifies anatomical brain changes, such as enlarged ventricles. Thus, although not diagnostic, this test can be used to identify anatomical changes in the brain, such as enlarged ventricles in a patient with suspected Alzheimer's disease. The correct answer is: CT

To identify anatomical changes in the brain of an 80-vear-old female patient with Alzheimer's disease, the most appropriate diagnostic technique is Select one: a. electroencephalogram (EEG) b. Glasgow Coma Scale c. amobarbitalsodium interview d. PET e. evoked EEG f. CT g. Folstein Mini-Mental State Examination

These signs (butterfly rash, fever), symptoms (fatigue, joint pain), and laboratory test results (mild anemia and presence of ANA) suggest that this patient has systemic lupus erythematosus (SLE), a connective tissue disorder. SLE is more common in African-American women of reproductive age and is exacerbated by exposure to the sun (such as this patient experienced on vacation). Personality changes and psychotic symptoms, such as the notion that people on television are referring to her (an idea of reference), also occur in connective tissue disorders such as SLE. Dissociative fugue,generalized anxiety disorder, brief psychotic disorder, and somatic symptom disorder are not diagnosed when a medical illness explains the behavioral and physical symptoms. The correct answer is: systemic lupus erythematosus (SLE)

Upon returning from a week-long vacation in the Caribbean, a 25-year-old African-American woman with no history of psychiatric symptoms seems agitated and anxious. When she tells her sisterthat a television newscaster is pub lie ally discussing her behavior, the sister brings her to the emergency department. The patient subsequently is admitted to the hospital with fever, fatigue, joint pain, and a rash across the bridge of her nose. Hematologic findings include mild anemia and presence of antinuclear antibodies (ANA). Of the following, the most likely explanation of the patients behavioral symptoms is Select one: a. systemic lupus erythematosus (SLE) b. brief psychotic disorder c. somatic symptom disorder d. dissociative amnesia with dissociative fugue e. generalized anxiety disorder

Damage to the right parietal lobe can result in impaired visual-spatial processing. This can lead to problems copying simple drawings and neglect of the left side as seen in this patient. The correct answer is: right parietal lobe

When a 70-year-old man who has had a stroke attempts to (a) divide a line in half, (b) turn single lines into "Xs," or (c) reproduce a clock face, he does the tasks like this (see Figure) effectively neglecting the left of the drawings. The area (s) of the brain most likely to be affected in this patient is (are) the Select one: a. hippocampus b. left frontal lobe c. amygdala d. right parietal lobe e. basal ganglia f. reticular system

Children begin to make marks (scribble) on paper at about 18 months of age The correct answer is: 16-30 months

When given a crayon, a child scribbles on paper. What is the age of the child? Select one: a. 12-15 months b. 4-6 months c. 7-11 months d. 0-3 months e. 16-30 months

In the United States, it is relatively uncommon to see a self-supporting adult, such as the 34-year-old medical resident, living with his parents. A 46-year-old man living with his wife and children is a common living situation in the United States; the divorce rate is high, but most people in their 40s are married, not single or divorced. It is also relatively common to see an 85-vear-old woman living with family members. The correct answer is: A 34-year-old medical resident living with his parents

Which of the following living situations is likely to be least common in the United States? Select one: a. A 34-year-old medical resident living with his parents b. An 85-year-old woman living with relatives c. A 46-year-old man living with his wife and children d. A 46-year-old single man living alone e. A 46-year-old divorced woman living with her 10-year-old son

The corpus callosum and the hippocampal, habenular, and anterior commissures connect the two hemispheres of the brain. The basal ganglia, reticular system, and amygdala do not have this function. The correct answer is: Hippocampal commissure and corpus callosum

Which of the following two structural entities connect the cerebral hemispheres? Select one: a. Basal ganglia and anterior commissure b. Anterior commissure and reticular system c. Hippocampal commissure and corpus callosum d. Reticular system and corpus callosum e. Amygdala and habenular commissure

The most likely diagnosis for this child is disinhibited social engagement disorder. Children with this disorder form indiscriminate attachments to strangers because their primary attachment figure, here the foster mother, does not interact normally with the child. Mild autism spectrum disorder and Rett's disorder are characterized by decreased, not increased, social interaction. The correct answer is: disinhibited social engagement disorder

A 2-year-old girl who has been in foster care since birth is very friendly and affectionate with strangers. She puts her arms out to them to be picked up and then 'cuddles up' to them. The foster mother states that the child has 'behavior problems' and then notes that she has never felt 'close' to the child. The most likely explanation for this child's behavior toward strangers is Select one: a. disinhibited social engagement disorder b. mild autism spectrum disorder c. Rett's disorder d. reactive attachment disorder e. typical behavior

Behavioral changes such as decreased impulse control, poor social behavior, and lack of characteristic modesty indicate that the area of the brain most likely to have been injured in this patient is the orbitofrontal cortex. Lesions of this brain area result in disinhibition, inappropriate behavior, and poor judgment. In contrast, lesions of the dorsolateral convexity of the frontal lobe result in decreased executive functioning (eg, motivation, concentration, and attention). The hypothalamus is associated with homeostatic mechanisms and the reticular system with consciousness and sleep. Damage to the amygdala results in decreased, not increased, aggression. The nucleus basalis of Meynert is a site of ACh production; its damage could result in deficits in intellectual function ing. The correct answer is: orbitofrontal cortex

A 24-year-old man sustains a head injury in an automobile accident. His father relates that prior to the accident, the patient was respectful, modest, controlled, and hard working. In the hospital, the patient is rude to the nurses and aides, loses his temper with the slightest provocation, and refuses to wear a hospital gown or anything else. These behavioral changes after the accident indicate that the area of the brain most likely to have been injured in this patient is the Select one: a. hypothalamus b. nucleus basalis of Meynert c. reticular system d. dorsolateral convexity of the frontal lobe e. amygdala f. orbitofrontal cortex

In sublimation, the surgeon reroutes his unconscious, unacceptable wish for committing a violent act to a socially acceptable route (cutting people during surgery). The correct answer is: Sublimation

A 26-year-old medical student who has unconscious angry, violent feelings chooses to do a surgery residency. Which of the following defense mechanisms is described in this scenario? Select one: a. Regression b. Reaction formation c. Sublimation d. Dissociation e. Displacement f. Projection g. Denial h. Rationalization i. Undoing j. Suppression k. Intellectualization l. Splitting

Dividing people or situations into categories of good and bad characterizes the defense mechanism of splitting. Splitting is commonly seen in people with borderline personality disorder. This disorder also is characterized by impulsive behavior such as making a suicide attempt for a trivial reason like that demonstrated in this example. Prejudice, chauvinism, and bias are more likely to be related to cultural factors than to the unconscious use of a defense mechanism. The correct answer is: splitting

A 28-year-old female patient is hospitalized after making a suicide attempt because her doctor did not respond to her offer to friend him on Facebook. When she is interviewed, the patient states that all female doctors are good but all male doctors are incompetent. The best explanation for this statement by the patient is Select one: a. bias b. splitting c. prejudice d. chauvinism e. lack of basic trust

This 4-year-old child is showing a typical reaction for his age. Children under the age of 6 years do not understand the finality of death and fully expect dead people to come back to life. That is why although he has been told that his father has died, this child repeatedly asks for his father. While he has been severely stressed, he is neither simply refusing to believe the truth nor showing delayed development. While it is possible that this boy has an undiagnosed head injury, a typical reaction is more likely. The correct answer is: a typical reaction for his age

A 4-year-old boy survives a house fire in which his father was killed. The child has no visible injuries and medical evaluation is unremarkable. Although he has been told that his father has died, in the weeks after the fire. The child continues to ask for his father. The best explanation for this boy's behavior is Select one: a. an undiagnosed head injury b. a typical reaction for his age c. refusal to believe the truth d. an acute reaction to severe stress e. delayed development

A 24-hour urine study is most likely to reveal elevated levels of VMA, a metabolite of norepinephrine. Anxiety, abdominal cramps and diarrhea, and skin flushing are symptoms of pheochromocytoma, a norepinephrine-secreting adrenal tumor. This picture is not seen with elevated levels of other neurotransmitter metabolites. The correct answer is: vanillylmandelic acid (VMA)

A 43-year-old man presents to the emergency department of a large hospital. He is very anxious and complains of abdominal cramps and diarrhea. The physician observes intense flushing of the man's skin. In this patient, a 24-hour urine study is most likely to reveal elevated levels of Select one: a. glycine b. 5-hydroxyindoleacetic acid (5-HIAA) c. acetylcholine d. vanillylmandelic acid (VMA) e. homovanillic acid (HVA)

Via classical conditioning, the patient has made an association between the sounds on the tape and sleeping, so she now falls asleep as soon as she hears the sounds. The correct answers are: classical conditioning, habituation

A 43-year-old woman is having difficulty falling asleep. Her physician advises her to listen to a 30-minute tape of ocean sounds and then go through a series of relaxation exercises every night prior to going to sleep. Two weeks later, the patient reports that she falls asleep as soon as she hears the sounds on the tape, even without doing the relaxation exercises. Falling asleep when she hears the tape is most likely to be due to which of the following? Select one: a. extinction b. habituation c. shaping d. punishment e. classical conditioning f. negative reinforcement g. positive reinforcement h. sensitization

This common clinical phenomenon is an example of classical conditioning. In this example, a woman comes into the hospital for an intravenous (IV) chemotherapy treatment (unconditioned stimulus). The chemotherapy drug istoxic and she becomes nauseated after the treatment (unconditioned response). The following month, when she enters the hospital lobby (conditioned stimulus), she becomes nauseated (conditioned response). Thus, the hospital where the treatments took place (conditioned stimulus) has become paired with chemotherapy (the unconditioned stimulus), which elicited nausea. Now, nausea (conditioned response) can be elicited by entering the hospital lobby (conditioned stimulus), even though she has not yet received the medication. In operant conditioning, behavior is learned by its consequences. Modeling is a type of observational learning. Shaping involves rewarding closer and closer approximations of the wanted behavior until the correct behavior is achieved. Extinction is the disappearance of a learned behavior when reinforcement is withheld. The correct answers are: classical conditioning, habituation

A 44-year-old woman has undergone three sessions of chemotherapy in a hospital. Each session has resulted in nausea. Before the fourth session, the patient becomes nauseated when she enters the hospital lobby. This patient's reaction is a result of the type of learning best described as Select one: a. punishment b. shaping c. classical conditioning d. sensitization e. positive reinforcement f. negative reinforcement g. habituation h. extinction

Of the listed brain areas, depression is most likely to be associated with damage to the left frontal lobe. The correct answer is: left frontal lobe

A 45-year-old male patient becomes depressed following a head injury. The area(s) of the brain most likely to be affected in this patient is (are) the Select one: a. hippocampus b. left frontal lobe c. basal ganglia d. reticular system e. !right parietal lobe f. amygdala

In reaction formation, the man denies his unconscious fear of flying and embraces the opposite idea by stating that he loves airplanes. The correct answer is: Reaction formation

A 45-year-old man who is unconsciously afraid of flying repeatedly states his love of airplanes. Which of the following defense mechanisms is described in this scenario? Select one: a. Sublimation b. Regression c. Denial d. Intellectualization e. Reaction formation f. Projection g. Undoing h. Rationalization i. Dissociation j. Splitting k. Suppression l. Displacement

While midlife is associated with the possession of power and authority, physical abilities decline. This time of life is also associated with a m id life crisis, which may include increased alcohol and drug use as well as an in creased likelihood of changes in social and work relationship. The correct answer is: Possession of power and authority

A 50-year-old male patient presents for an insurance physical. Which of the following developmental signposts is most likely to characterize this man? Select one: a. Peak physical development b. Strong resistance to changes in social relationships c. Possession of power and authority d. Strong resistance to changes in work relationships e. Decreased alcohol use

Using denial, this patient has seemingly forgotten an aspect of external reality, that is. the letter about his problematic PSA test. The correct answer is: Denial

A 52-year-old man receives a letter from his physician informing him that his level of prostate-specific antigen (PSA) was abnormally high during his last visit. When the man appears at his physician's office for a follow-up visit, he complains about a headache but does not mention or seem to remember receiving the letter about his PSA test. Which of the following defense mechanisms is described in this scenario? Select one: a. Rationalization b. Splitting c. Regression d. Denial e. Reaction formation f. Projection g. Undoing h. Suppression i. Sublimation j. Dissociation k. Displacement l. Intellectualization

These 52-year-old women in good general health are going through menopause. The most common symptom of menopause occurring cross-culturally hot flashes, a purely physiological phenomenon. In most women, menopause is not characterized by psychopathology such as severe depression or anxiety or physical symptoms like fatigue and lethargy. The correct answer is: Hot flashes

A 52-year-old woman in the United States has a 52-year-old female friend in Australia. Both are in good general health and neither has menstruated for about year. Which of the following symptoms are both women most likely to experience at this time? Select one: a. Fatigue b. Lethargy c. Severe anxiety d. Severe depression e. Hot flashes

Although neuroimaging cannot be used to diagnose psychiatric disorders, brains of patients with schizophrenia such as this woman are likely to show decreased volume of limbic structures such as the hippocampus; increased size of cerebral ventricles due, in part, to brain shrinkage; and decreased glucose utilization in the frontal cortex. The correct answer is: decreased, increased, decreased

A 55-year-old woman was diagnosed with schizophrenia at the age of 22. If this diagnosis was appropriate, the volume of the hippocampus, the size of the cerebral ventricles, and glucose utilization in the frontal cortex of this patient are now most likely to be, respectively Select one: a. increased, decreased, increased b. decreased, decreased, increased c. decreased, increased, decreased d. decreased, decreased, decreased e. increased, in creased, increased

Chromosome 16 and chromosome 9 are both associated with tuberous sclerosis. Seizures, cognitive defects, autistic behavior, and forehead plaques in this 6-year-old child are seen in this disorder. The correct answer is: 16

A 6-year-old child shows seizures, cognitive defects, and autistic behavior. The child also shows raised discolored areas on her forehead (forehead plaques). Which of the following chromosomes is most likely to be involved in the etiology of this child's symptoms? Select one: a. X b. 21 c. 16 d. 18 e. 1

This patient is showing evidence of Alzheimer's disease. Of the listed brain areas, the major one implicated in Alzheimer's disease is the hippocampus. The correct answer is: hippocampus

A 69-year-old former bank president cannot tell you the name of the current president and has difficulty identifying the woman sitting next to him (his wife). He began having memory problems 3 years ago. Atrophy of which area(s) of the brain is(are) most likely to be seen in this patient? Select one: a. amygdala b. hippocampus c. left frontal lobe d. basal ganglia e. right parietal lobe f. reticular system

The therapeutic action of memantine in Alzheimer's disease is believed to be to decrease the in flux of glutamate, ultimately blocking the influx of calcium, which can lead to nerve cell degeneration and death. In contrast to a group of drugs also used to treat Alzheimer's, that is, the acetylcholinesterase inhibitors, memantine does not directly affect acetylcholine. The correct answer is: To block the in flux of calcium

A 72-year-old man with Alzheimer's disease is being treated with memantine. What is believed to be the basis of the therapeutic action of memantine on neurons in the brain? Select one: a. To inhibit the action of acetylcholine b. To facilitate the in flux of calcium c. To block the in flux of calcium d. To increase the in flux of glutamate e. To inhibit the action of acetylcholinesterase

Preschool children usually cannot comprehend the meaning of death and commonly believe that the dead person or pet will come back to life. Children over the age of 6 years commonly are aware of the finality of death. The correct answer is: 4 years

A child's pet has recently died. The child believes that the pet will soon come back to life. This child is most likely to be of age Select one: a. 4 years b. 9 years c. 7 years d. 6 years e. 11 years

Because the behavior (hitting the dog) decreased, the scolding that this child received is probably punishment. Both negative and positive reinforcement increase behavior. Shaping involves rewarding closer and closer approximations of the wanted behavior until the correct behavior is achieved. In classical conditioning, a natural or reflexive response (behavior) is elicited by a learned stimulus (a cue from an internal orexternal event). The correct answer is: punishment

A father scolds his child when she hits the dog. The child stops hitting the dog. This change in the child's behavior is most likely to be a result of Select one: a. positive reinforcement b. habituation c. sensitization d. shaping e. extinction f. classical conditioning g. punishment h. negative reinforcement

The doctor should remind the parents to pay more attention to the older child. The child is likely to be frightened by his younger sibling's illness and the attitudes of his parents toward the younger child. School-age children such as this one may become withdrawn or act out by showing bad behavior when fearful or depressed. While he can be included in the care of his brother, it is not appropriate to insist that he take more responsibility for him. Ignoring his behavior or punishing him can increase his fear and withdrawal. False reassurance such as telling the child that everything will be fine is not appropriate. The correct answer is: pay more attention to him

A formerly outgoing 10-year-old boy begins to do poorly in school after his 6-year old brother is diagnosed with leukemia. He now prefers to watch television alone in his room and does not want to socialize with his friends. His parents are very stressed by caring for the younger child but do not ask the older child for help. The most appropriate suggestion for the doctor to make with respect to the 10-year-old is to tell the parents to Select one: a. pay more attention to him b. insist that he take more responsibility for caring for his younger brother c. remove the television from his room d. tell him not to worry, everything will be fine e. ignore his behavior

Latency-age children (age 7-11 years) have little interest in those of the opposite sex and often criticize or avoid them. In contrast, younger children do not show strong gender preferences for playmates, and teen agers comm only seek the company of opposite-sex peers. The correct answer is: 9 years

A girl tells her mother that she hates the boys because they are noisy and stupid. The age of this girl is most likely to be Select one: a. 4 years b. 6 years c. 9 years d. 13 years e. 15 years

The students'response to the fire alarm is most likely to have been learned by habituation, that is desensitization. In this form of learning, continued exposure to a stimulus (the fire alarm, in this example) results in a decreased response to the stimulus. Thus, while the students respond quickly to the fire alarm at first, with repeated soundings of the alarm, they ultimately fail to respond to it. If sensitization had occurred, the students would have responded more quickly with each exposure to the alarm. In classical conditioning, a natural response is elicited by a learned stimulus.In operant conditioning, reinforcement is a consequence of a behavior that alters the likelihood that the behavior will occur again. Punishment is the introduction of an aversive stimulus that reduces the rate of an unwanted behavior, while extinction is the disappearance of a learned behavior when reinforcement is withheld. The correct answer is: habituation

A grade school principal has 1 week to try out a new fire-alarm system for the school. He decides to test the system three times during the week. The first time the alarm is sounded, all of the students leave the schoolwithin 5 minutes. The second time, it takes the students 15 minutes to leave the school. The third time the alarm is sounded, the students ignore it. The students'response to the fire alarm the third time it is sounded is most likely to have been learned by Select one: a. sensitization b. fixed ratio reinforcement c. continuous reinforcement d. punishment e. habituation f. variable ratio reinforcement g. classical conditioning

This man is using the partly conscious defense mechanism of suppression during the time that he is arranging for his work to be done by someone else before going to the hospital. The correct answer is: Suppression

A man who has just received word that his child has been in an accident and has been taken to the hospital calmly arranges for his work to be done by a colleague before he rushes to the hospital. Which of the following defense mechanisms is described in this scenario? Select one: a. Dissociation b. Undoing c. Rationalization d. Displacement e. Suppression f. Sublimation g. Intellectualization h. Reaction formation i. Splitting j. Denial k. Projection l. Regression

The doctor who becomes very angry at her patient for not taking his medication is showing a countertransference reaction. This excessive show of emotion is a result of re-experiencing feelings about her son's behavior in her relationship with the noncompliant patient. It is important for the doctor to identify this reaction because it can interfere with her medical judgment. This doctors reaction to the patient is less likely to be related to dislike or fear of the patient. The correct answer is: countertransference

A physician becomes very angry with a patient when the patient does not take his medication. The patient reminds the doctor of her rebellious son. This physician's intense reaction to the patient's behavior is most likely to be a result of Select one: a. countertransference b. fear of the patient c. negative transference d. positive transference e. dislike of the patient

Teenagers who become pregnant frequently are depressed. Come from homes where the parents are divorced, have problems in school, and may not know about effective contraceptive methods. Studies have not indicated that living in a rural area is related to teenage pregnancy The correct answer is: Depressed mood

A physician discovers that a 15-year-old patient is pregnant. Which of the following factors is likely to have contributed most to her risk of pregnancy? Select one: a. Depressed mood b. High achievement in school c. Living in a rural area d. Intact parental unit e. Having received information about contraceptive methods

The formation of a personal identity is usually achieved during the teenage years. The concepts of seriation and conservation and an understanding of the concept of "fair play" are gained during the school-age years. Parallel play is usually seen between ages 2 and 4 years. The correct answer is: The formation of a personal identity

A physician is asked to evaluate the development of an 11-year-old girl. Which of the following milestones is usually not acquired until after the age of 11 years? Select one: a. An understanding of the concept of "fair play" b. Parallel play c. The formation of a personal identity d. The concept of seriation e. The concept of conservation

When com pared to younger ages, peers and non familial adults become more important to the latency-age child and the family becomes less important. Children 7-11 years of age have the capacity for logical thought, have a conscience, identify with the same-sex parent, and show a strong preference for playmates of their own sex. The correct answer is: Relatively stronger importance of friends over family when compared to children of younger ages

A physician is conducting a school physical on a typical 10-year-old girl. When interviewing the child, the physician is most likely to find which of the following psychological characteristics? Select one: a. Lack of conscience formation b. Poor capacity for logical thought c. Identification with her father d. Relatively stronger importance of friends over family when compared to children of younger ages e. No preference with respect to the sex of playmates

Increased body fluid level of homovanillic acid (HVA), a major metabolite of dopamine, is seen in schizophrenia. Decreased HVA is seen in Parkinson's disease, depression, and in medicated schizophrenic patients. Increased vanillylmandelic acid (VMA), a metabolite of norepinephrine, is seen in pheochromocytoma. Decreased body fluid level of 5-HIAA, a metabolite of serotonin, is seen in depression and in bulimia. The correct answer is: Schizophrenia

Analysis of the blood plasma of a 45-year-old m ale patient shows increased concentration of homovanillic acid (HVA). This elevation is most likely to be associated with which of the following conditions? Select one: a. Depression b. Pheochromocytoma c. Bulimia d. Parkinson's disease e. Schizophrenia

This child is most likely to be 4 years of age. Preschool children do not yet understand the concept of conservation (ie, that the quantity of a substance remains the same regardless of the shape that it is in). Thus, this child believes that a hot dog cut in to three pieces has more in it than when it was in only one piece. Children understand this concept better as they approach school age. The correct answer is: 4 vears

At the lunch table a child asks his mother to cut his hot dog up into three pieces so that he can have three times as much to eat. The age of this child is most likely to be Select one: a. 13years b. 4 vears c. 15 years d. 6 years e. 9 years

Children begin to show social smiling between 1 and 2 months of age. The correct answer is: 0-3 months

At what age child can smile in response to a human face? Select one: a. 0-3 months b. 12-15 months c. 16-30 months d. 4-6 months e. 7-11 months

Transferring objects from hand to hand commonly occurs at about 10 months of age. The correct answer is: 7-11 months

At what age child transfers toys from one hand to the other? Select one: a. 16-30 months b. 0-3 months c. 4-6 months d. 12-15 months e. 7-11 months

Gamma-aminobutyric acid (GABA) is an inhibitory amino acid neurotransmitter in the CNS. Thus, the activity of GABA in the brain of this anxious patient is likely to be decreased. Decreased serotonin and increased dopamine are also involved in anxiety. The correct answer is: decreased

Avery anxious 25-year-old patient is examined in the emergency room. There is no evidence of physical illness. If it could be measured, the gamma-aminobutyric acid (GABA) activity in the brain of this patient would most likely be Select one: a. higher than the activity of dopamine b. unchanged c. decreased d. higher than the activity of serotonin e. increased

The major objective of hearing loss screening in newborns is for early diagnosis and treatment of hearing loss in order to prevent language development delay. In older children, evaluation of hearing loss is useful in determine the necessity of using cochlear implants or speech therapy. Hearing loss is not specifically associated with motor development delay. The correct answer is: diagnose and treat hearing loss early in order to prevent language development delay

In a major city hospital, the hearing of all newborns is evaluated shortly after birth. The major objective of this hearing loss screening is to Select one: a. determine the necessity of speech therapy before the age of 1 year b. diagnose and treat hearing loss early in order to prevent motor development delay c. increase the cost-effectiveness of treatment for hearing loss d. diagnose and treat hearing loss early in order to prevent language development delay e. determine the necessity of using cochlear implants before the age of 6 months

This child has received money on unpredictable occasions for cleaning his room. Behavior learned in this way (ie, by variable ratio reinforcement) is very resistant to extinction and continues even when it is not rewarded. Behavior learned by fixed schedules of reinforcement (ratio or interval) is less resistant to extinction. Behavior learned by continuous reinforcement is least resistant to extinction. Punishment is aversive and is aimed at suppressing an undesirable behavior. The correct answer is: Variable ratio reinforcement

In the past, a child has on occasion received money for cleaning his room. Despite the fact that he has not received money for cleaning his room for the past month, the child's room-cleaning behavior continues (is resistant to extinction). This child's room-cleaning behavior was probably learned using which of the followingmethods? Select one: a. Variable ratio reinforcement b. Fixed ratio reinforcement c. Fixed interval reinforcement d. Continuous reinforcement e. Punishment

The parents should be reassured that like their child. 13-month-old children typically say only a few words and are just starting to walk. Children typically show no interest in nor can they be toilet trained until they are at least 2.5-3 years of age. The correct answer is: reassure the parents that the child's behavior is typical for her age

Parents of a 13-month-old child tell the doctor that the child shows no interest in toilet training. They also relate that the child speaks about 10 words and has just started to walk unassisted. The doctor should Select one: a. refer the family to a pediatric gastroenterologist b. reassure the parents that the child's behavior is typical for her age c. contact child protective services d. tell the parents that the child's hearing should be checked as soon as possible e. evaluate the child for delayed motor development

The patient who becomes very angry at his new doctor is showing a negative transference reaction. This emotional demonstration is likely to be a result of re-experiencing negative feelings about his relationship with his middle-aged father in his relationship with the middle-aged male doctor. In negative transference, patients become resentful or angry toward the doctor if their desires and expectations are not realized. Thismay lead to noncompliance with medical advice. In positive transference, patients have a high level of confidence in the doctor. Patients may also overidealize or develop sexual feelings toward the doctor. This patient's reaction to the new doctor is less likely to be related to dislike or fear of the doctor. The correct answer is: Negative transference

When a 27-year-old patient who had a contentious relationship with his father joins a new health insurance plan, he must change from his primary care physician, a young man, to a new physician, a middle-aged man. On his first visit to the new doctor, the patient seems annoyed with everything the doctor says and states. 'You are an old man with old-fashioned ideas: you just want to control my life.' This patient's behavior is most closely related to which of the following? Select one: a. Dislike of the doctor b. Negative transference c. Positive transference d. Fear of the doctor e. Countertransference

At the age of 3 years, the child can ride a tricycle, copy a circle, and engage in parallel play (play alongside but not cooperatively with other children). However. 3-year-old children such as this one should have a vocabulary of about 900 words and speak in complete sentences. The correct answer is: typical, typical, needs evaluation

When a physician conducts a well-child check up on a 3-year-old boy, he finds that the child can ride a tricycle, copy a circle, engage in parallel play with other children, name some of his body parts (e.g., nose, eyes) but not others (eg, hand, finger), and has about a 50-word vocabulary. With respect to motor, social, and cognitive/verbal skills, respectively, the best description of this child is Select one: a. typical, typical, needs evaluation b. needs evaluation, typical, needs evaluation c. typical, needs evaluation, typical d. typical, typical, typical e. typical, needs evaluation, needs evaluation

A Two-year-old children speak in two-word sentences (eg, "Me go"). Toilet training or the ability to spend most of the day away from the mother does not usually occur until age 3. Children engage in cooperative play starting at about age 4 and can ride a three-wheeled bicycle at about age 3. The correct answer is: Speaks in two-word sentences

When a physician conducts a well-child check up on a typical 2-year-old girl, the child is most likely to show which of the following skills or characteristics? Select one: a. Engages in cooperative play b. Can ride a tricycle c. Speaks in two-word sentences d. Can comfortably spend most of the day a way from her mother e. Is toilet trained

Primary process thinking is associated with pleasure seeking, disregards logic and reality, has no concept of time, and is not accessible to the conscious mind. Secondary process thinking is logical and is associated with reality. The correct answer is: is associated with pleasure seeking

When having a manic episode, a 53-year-old patient with bipolar disorder shows primary process thinking. This type of thinking Select one: a. is associated with pleasure seeking b. is closely attuned to time c. is associated with reality d. is accessible to the conscious mind e. is logical

The patient's elevated blood pressure in the doctor's office is the conditioned (learned) response. This response results from an association that has been made by classical conditioning between the doctor and/or his white coat (conditioned stimulus) and something negative in the patient's past (unconditioned stimulus), a reaction commonly called "white-coat hypertension." The cue that this response is learned is that the patient's blood pressure is relatively normal when taken at home. The correct answer is: the conditioned response

Whenever a 46-year-old man visits his physician, his blood pressure is elevated. When the patient takes his own blood pressure at home, it is usually normal. The doctor says that while other tests need to be done, the patient is probably showing "white-coat hypertension." For this scenario, the patients blood pressure in the doctor's off ce represents Select one: a. the unconditioned response b. the conditioned response c. the unconditioned stimulus d. the conditioned stimulus

The id is present at birth, the ego begins to develop immediately after birth, and the superego begins to develop at about age 6 years. The correct answer is: The id and the ego only

Which of the following structures of the mind is (are) at least partly developed in a typical 4-year-old child? Select one: a. Not the id, ego, or superego b. The id, ego, and superego c. The ego and superego only d. The id and the ego only e. The id only

In Freud's structural theory, the mind is divided into the id. ego. and superego. The id operates completely on an unconscious level, while the ego and superego operate partly on an unconscious and partly on preconscious and conscious levels. The correct answer is: The id, ego, and superego

Which of the following structures of the mind work at least partly on an unconscious level? Select one: a. The id only b. Not the id, ego, or superego c. The id and the ego only d. The ego and superego only e. The id, ego, and superego

The best description of this girls behavior is typical. Her desire to sleep with her mother is a sign of regression, a defense mechanism that is common in typical children under stress. Because she continues to play well when away from her mother, this is not separation anxiety disorder. There is also no evidence of delayed development, lack of basic trust, or ADHD. The correct answer is: typical behavior with regression

While she previously slept in her own bed. after her parents' divorce, a 5-year-old girl begs to be allowed to sleep in her mother's bed every night. She says that a "robber" is under her bed. She continues to do well in kindergarten and to play with her friends. The best description of this girl behavior is Select one: a. typical behavior with regression b. delayed development c. separation anxiety disorder d. lack of basic trust e. ADHD

The physician should talk to this 15-year-old girl alone. In addition to sexual and drug use issues, those that involve body image such as body weight ideally should be discussed with a teen ager alone, without other family members present. The correct answer is: talk to the girl alone

A 15-year-old overweight girl and her mother come to see the doctor for advice about diet and exercise. The mother states that she does not know why the girl is overweight because she cooks the same food or her and her slim 16-year-old brother. The doctor should first Select one: a. talk to the mother, the brother, and the girl together b. talk to both the teens with the mother present c. talk to the girl with the mother present d. talk to the mother alone e. talk to the girl alone

Mildly and moderately intellectually disabled children are aware that they have a developmental delay. They often have low self-esteem and may become socially withdrawn. In part, these problems occur because they have difficulty communicating with and competing with peers. The correct answer is: knows that she is developmentally delayed

A 10-year-old girl with Down's syndrome and an IQ of 60 is brought to the physician's off ce for a school physical. When the doctor interviews this girl, he is most likely to find that she Select one: a. is socially outgoing b. competes successfully with peers c. knows that she is developmentally delayed d. communicates well with peers e. has good self-esteem

Peer pressure has a major influence on the behavior of adolescents who tend to do what other adolescents are doing. Depression, the smoking behavior of the parents, and the addictive quality of cigarettes have less of an influence. Most teen agers have been educated with respect to the dangers of smoking. The correct answer is: his peers smoke

A 15-year-old boy tells his physician that he has been smoking cigarettes for the past year. He relates that his friends smoke and his father smokes. The most likely reason that this teenager does not attempt to stop smoking is because Select one: a. smoking is addictive b. he is depressed c. his father smokes d. he does not know that smoking is harmful e. his peers smoke

Assaultive, impulsive, aggressive behavior like that seen in this 28-year-old male patient is associated with decreased levels of serotonin in the brain. Levels of 5-HIAA (5-hydroxyindoleacetic acid), the major metabolite of serotonin, have been shown to be decreased in the body fluids of violent, aggressive, impulsive individuals as well as depressed individuals. MHPG (3-methoxy-4-hydroxyphenylglycol), a metabolite of norepinephrine, is decreased in severe depression, while homovanillic acid (HVA), a metabolite of dopamine, is decreased in Parkinson's disease and depression. The correct answer is: decreased 5-HIAA

A 28-year-old male patient is brought to the emergency room after a fight in which he attacked a man who cut into his line at the supermarket checkout. In the emergency room, he remains assaultive and combative. The body fluids of this patient are most likely to show Select one: a. decreased homovanillic acid (HVA) b. increased 5-hydroxyindoleacetic acid (5-HIAA) c. decreased MHPG d. increased 3-methoxy-4-hydroxyphenylglycol (MHPG) e. decreased 5-HIAA

The resident's behavior in dealing with this patient reflects his use of the defense mechanism of intellectualization. The resident has used his technical knowledge to avoid experiencing the emotion associated with telling the patient that she is dying. The correct answer is: Intellectualization

A 28-year-old medical resident is assigned to tell a patient that her illness is terminal. Prior to seeing the patient, the resident conducts extensive library research on the details and statistics of length of survival of people with this illness. When he speaks to the patient, he cites the journal articles that he has read, including a detailed explanation of the theories of the etiology of her condition. Later that day, the resident tells the attending physician that the patient did not seem to understand what he told her. Which of the following defense mechanisms is described in this scenario? Select one: a. Regression b. Sublimation c. Reaction formation d. Denial e. Suppression f. Rationalization g. Projection h. Splitting i. Undoing j. Dissociation k. Displacement l. Intellectualization

The effectiveness of clonidine in treating withdrawal symptoms associated with the use of opioids and sedatives is believed to be due to its action on alpha 2-adrenergic receptors, for example, reducing the firing rate of noradrenergic neurons, most of which are located in the locus ceruleus. The correct answer is: locus ceruleus

A 30-year-old woman who is withdrawing from heroin shows intense anxiety, increased pulse, elevated blood pressure, and a hand tremor. Her symptoms improve when she is given clonidine, an alpha 2-adrenergic receptor agonist.The area (s) of the brain most likely to be involved in the improvement in this patient's symptoms is (are) the Select one: a. right parietal lobe b. locus ceruleus c. raphe nuclei d. substantia nigra e. amygdala f. basal ganglia

This woman who bakes cookies for her abusive father is managing her hostility toward him by using the defense mechanism of reaction formation. In this defense mechanism, a person adopts behavior that is opposite to the way she really feels, that is, this woman feels intense anger toward her father but shows caring behavior toward him. The correct answer is: Reaction formation

A 30-year-old woman who was abused by her father throughout her childhood manages her hostility toward him by baking cookies for him. Which of the following defense mechanisms is described in this scenario? Select one: a. Splitting b. Rationalization c. Dissociation d. Regression e. Projection f. Denial g. Suppression h. Displacement i. Reaction formation j. Intellectualization k. Sublimation l. Undoing

Using projection, the husband attributes his own unconscious, unacceptable sexual feelings toward another woman to his wife. The correct answer is: Projection

A 32-year-old man who is unconsciously attracted to his wife's sister becomes extremely jealous whenever his wife speaks to another man. Which of the following defense mechanisms is described in this scenario? Select one: a. Rationalization b. Denial c. Intellectualization d. Suppression e. Splitting f. Displacement g. Sublimation h. Undoing i. Regression j. Projection k. Reaction formation l. Dissociation

This patient who relates that she wakes up fully dressed at least twice a week and receives phone calls from men whom she does not remember meeting is exhibiting dissociative identity disorder (multiple personality disorder). Dissociation, separating part of one's consciousness from real life events, is the defense mechanism used by individuals with this disorder. It is likely that this patient met the men who have her phone number but does not remember meeting them because at that time she was showing another personality. The correct answer is: Dissociation

A 34-year-old woman relates that she wakes up fully dressed at least twice a week but then is tired all day. She also notes that she frequently receives phone calls from men who say they met her in a bar but whom she does not remember meeting. Which of the following defense mechanisms is described in this scenario? Select one: a. Dissociation b. Intellectualization c. Reaction formation d. Suppression e. Rationalization f. Sublimation g. Displacement h. Splitting i. Denial j. Regression k. Projection l. Undoing

Sleep-arousal mechanisms are affected by damage to the reticular system. The correct answer is: reticular system

A 35-year-old fem ale patient reports that she has difficulty sleeping ever since she sustained a concussion in a subway accident. The area (s) of the brain most likely to be affected in this patient is (are) the Select one: a. hippocampus b. left frontal lobe c. right parietal lobe d. basal ganglia e. reticular system f. amygdala

Regression, going back to a less mature way of behaving, is the defense mechanism used by this woman scheduled for surgery the next day who insists that her mother stay overnight in the hospital with her. The correct answer is: Regression

A 35-year-old lawyer scheduled for surgery the next day insists that her mother stay overnight in the hospital with her. Which of the following defense mechanisms is described in this scenario? Select one: a. Intellectualization b. Dissociation c. Reaction formation d. Denial e. Splitting f. Regression g. Rationalization h. Projection i. Displacement j. Undoing k. Sublimation l. Suppression

The useful employment in her abstract art of this woman's 'messy' tendencies is an example of the defense mechanism of sublimation. The correct answer is: Sublimation

A woman, whose parents and teachers complained about how messy she was as a child, grows up to become a famous abstract painter. Her technique involves throwing paint and small objects at large canvases and then using her fingers to mix the colors and textures. Which of the following defense mechanisms is described in this scenario? Select one: a. Suppression b. Denial c. Projection d. Dissociation e. Intellectualization f. Regression g. Splitting h. Sublimation i. Rationalization j. Displacement k. Reaction formation l. Undoing

The best thing for the medical student to do at this time is to interact with the child. Since they do not speak the same language, involving children of this age in an interactive activity such as using the stethoscope or drawing pictures together is the best choice here. Neither giving the child a toy nor looking in her ears is an interactive activity. The student, not the nurse, is responsible for the child in this instance Sedation is in appropriate at this time; social activity is often effective in decreasing a patients anxiety The correct answer is: listen to the child's heart with a stethoscope and then let the child try using the stethoscope to listen to his heart

A medical student on a surgery rotation is assigned to stay with a 9-year-old girl who is waiting to have surgery to repair a cleft palate. The girl, who has recently arrived alone from Laos, does not speak English and appears anxious. The hospital administrator has requested a translator who has not yet arrived. At this time, the most appropriate action for the medical student to take is to Select one: a. listen to the child's heart with a stethoscope and then let the child try using the stethoscope to listen to his heart b. sedate the child to decrease her anxiety c. suggest that the nurse stay with the child so that he can review her chart d. give the child a toy to keep her occupied e. look in the child's ears with an otoscope

The social smile (smiling in response to seeing a human face) is one of the first developmental milestones to appear in the infant and is present by 1-2 months of age. Stranger anxiety (fear of unfamiliar people) appears at about 7 months of age and indicates that the infant has a specific attachment to the mother. Rapprochement (the tendency to run away from the mother and then run back for comfort and reassurance) appears at about 18 months of age. Core gender identity (the sense of self as male or female) is established between 2 and 3 years of age. Transient phobias (irrational fears) occur in typical children, appearing most commonly at 4-5 years of age. The correct answer is: Social smile

A mother brings her 4-month-old child to the pediatrician for a well-baby examination. Which of the following developmental signposts can the doctor expect to be present in this infant if the child is developing typically? Select one: a. Social smile b. Phobias c. Stranger anxiety d. Rapprochement e. Core gender identity

In this example, the child's crying behavior increases as a result of positive reinforcement, being picked up by his mother each time he cries. The mother's behavior (picking up the child) increases as a result of negative reinforcement; she picks him up to avoid hearing him cry. The correct answer is: negative reinforcement

A mother picks up her 3-month-old baby each time he cries. The child cries on more and more occasions each day and the mother picks him up more frequently. The mother has learned to pick up the child more frequently mainly by the process of Select one: a. classical conditioning b. punishment c. sensitization d. negative reinforcement e. habituation f. extinction g. positive reinforcement h. shaping

In this example, the child's crying behavior increases as a result of positive reinforcement, being picked up by his mother each time he cries. The mother's behavior (picking up the child) increases as a result of negative reinforcement; she picks him up to avoid hearing him cry. The correct answer is: positive reinforcement

A mother picks up her 3-month-old baby each time he cries. The child cries on more and more occasions each day and the mother picks him up more frequently. This child has learned to get picked up mainly by the process of Select one: a. punishment b. negative reinforcement c. extinction d. sensitization e. positive reinforcement f. classical conditioning g. shaping h. habituation

The age of this woman's son is most likely to be 15 years. Middle adolescents (15-17 years) often challenge parental authority and have feelings of omnipotence (eg, nothing bad will happen to them because they are all powerful). Younger adolescents (11-14 years) are unlikely to challenge parental rules and authority. Older adolescents (18-20 years) have developed self-control and a more realistic picture of their own abilities. The correct answer is: 15 years

A mother tells the physician that she is concerned about her son because he consistently engages in behavior that is dangerous and potentially life threatening. The age of her son is most likely to be Select one: a. 13 years b. 15 years c. 11 years d. 18 years e. 20 years

Although this boy is probably showing typical behavior for a 6-year-old, the doctor needs to know more about this mothers concerns regarding her son. Since he is doing well in school; there is no need to speak to the teacher or the child. Simply saying "he must be hard to handle or he is fine, do not worry" will not address the mothers concerns. The correct answer is: say "Tell me more about your concerns regarding your son"

A mother worriedly reports that her 7-year-old son is often dirty when he comes in from playing. She notes that he digs in the dirt, wipes his face with his dirty hands, and climbs trees outside of the home. She states that she is worried that he will catch a disease or injure him self. The mother also reports that she had a meeting with the child's teacher who told her that the child is doing well in school. The next step in management is for the doctor to Select one: a. speak with the child b. say "Tell me more about your concerns regarding your son" c. say "He must be hard to handle" d. speak with the child's teacher e. say "He is fine, do not worry"

This patient with obsessive-compulsive disorder is using the defense mechanism of undoing. Counting the lights removes or "undoes" the student's anxiety, which is likely to be related to his school performance. The correct answer is: Undoing

A patient who has been diagnosed with obsessive-compulsive disorder tells the doctor that he has to count all the lights in the ceiling before he can sit down to study. If he does not count the lights, he becomes anxious and is unable to study. Which of the following defense mechanisms is described in this scenario? Select one: a. Regression b. Intellectualization c. Denial d. Reaction formation e. Sublimation f. Suppression g. Displacement h. Projection i. Splitting j. Undoing k. Dissociation l. Rationalization

Because the behavior (exercise) is increased to avoid something negative (insulin injections), this is an example of negative reinforcement. The correct answer is: negative reinforcement

A patient with diabetes increases her time spent exercising in order to reduce the number of insulin injections she must receive. The increased exercising behavior most likely to be a result of Select one: a. extinction b. negative reinforcement c. classical conditioning d. habituation e. shaping f. punishment g. sensitization h. positive reinforcement

Parents should be present when a physician speaks to a younger child, but teenagers usually should be interviewed particularly about sexual issues, without parents present. Thus, the doctor should ask the 15-year-old to leave and talk to the 8-year old with the mother present. Then, the doctor should talk to the 15-year-old alone. The correct answer is: ask the 15-year-old to leave and talk to the 8-year-old with the mother present and then, talk to the 15-year-old alone

A physician is scheduled to see an 8-year-old and a 15-year-old sisters for routine checkups. They had consecutive appointments, but when the doctor enters the exam in ing room, they are both there with their mother. Most appropriately the doctor should Select one: a. ask the mother and the older girl to leave. Talk to the younger child alone and then talk to the older girl alone b. ask both girls to leave, talk to the mother alone, and then ask the mother to come back in and talk to all three together c. ask the mother to leave, talk to both girls together and then talk to the mother alone d. ask both girls to leave and talk to the mother alone and then, ask the mother to leave and talk to the two girls together e. ask the 15-year-old to leave and talk to the 8-year-old with the mother present and then, talk to the 15-year-old alone

Statements such as "I can't stop smoking because I'll gain weight" or "when I'm sick. I only want to eat junk food" are examples of the defense mechanisms of rationalization and regression, respectively. In rationalization, a person distorts her perception of an event so that its negative outcome seems reasonable, for example, because she feels unable to stop smoking, this patient claims (and so she reasonably feels) thatgaining weight is worse than smoking, a life-threatening habit. In regression, ill patients revert to behavior patterns like those seen in someone of a younger age (eg, eating junk food, crying). Defense mechanisms such as these are unconscious mental techniques that decrease anxiety and help people to maintain a sense of equilibrium and self-esteem. The correct answer is: are examples of the use of defense mechanisms

A primary care physician notices that many of her patients use statements like "I can't stop smoking because I'll gain weight" or "when I'm sick, I only want to eat junk food." Statements like these Select one: a. decrease a patient's sense of self-esteem b. are examples of the use of defense mechanisms c. are conscious mental techniques d. increase anxiety e. produce conflict in the conscious mind

Stranger anxiety (the tendency to cry and withdraw in the presence of an unfamiliar person) develops in typical infants at 7-9 months of age. It does not indicate that the child is developmentally delayed, emotionally disturbed, or that the child has been abused but rather that the child can now distinguish familiar from unfamiliar people. Stranger anxiety is more common in children who are cared for by only one person and is reduced in those exposed to many different caregivers. The correct answer is: withdrawal from the doctor

A typical 8-month-old child is brought to the pediatrician for his monthly well-baby examination. The child is the family's first, and he is cared for at home by his mother. When the doctor approaches the child in his mother's arms, the child's behavior is most likely to be characterized by Select one: a. smiling at the doctor b. withdrawal from both the doctor and the mother c. indifference to the doctor d. withdrawal from the doctor e. an anticipatory posture toward the doctor (arms held out to be picked up)

Postpartum blues may occur in one-third to one-half of new mothers and can last up to 2 weeks. Intervention involves support and practical help with the child. Brief psychotic disorder is rare, occurring in less than 1% of new mothers and lasting up to 1 month after childbirth. Postpartum depression occurs in 5%-10% of new mothers and is treated primarily with antidepressant medication. The correct answer is: Postpartum blues will usually last up to 2 weeks.

A well-trained, highly qualified obstetrician has a busy practice. Which of the following is most likely to be true about postpartum reactions in this doctor's patients? Select one: a. Brief psychotic disorder will usually last about 1 year. b. Postpartum blues will occur in about 10% of the patients. c. Major depression will occur in about 25% of the patients. d. Brief psychotic disorder will occur in about 8% of the patients. e. Postpartum blues will usually last up to 2 weeks.

The most appropriate action for the physician to take at this time is to ask to speak to the girl alone. The physician can then ask the girl about her sexual activity and provide contraceptives and counseling if she wishes, without notification or con sent from the mother. The mother's wishes in this circumstance are not relevant to the physicians action; the teenager is the patient. The correct answer is: ask to speak to the girl alone

A woman and her 15-year-old daughter present to the physician's office together. The mother asks the physician to ft her daughter for a diaphragm. The most appropriate action for the physician to take at this time is to Select one: a. ask the mother why she wants a diaphragm for her daughter b. recommend that the girl see a counselor c. ask to speak to the girl alone d. follow the mothers wishes e. ask the girl if she is sexually active

"Did you experience any emotional difficulties after the birth of your other children?" is an important question since a predictor of postpartum reactions is whether or not they have occurred before. This patient is probably worried because she has had previous problems. Reassuring statements, such as "Most women who worry about depression never experience it," "Do not worry, there are many effective medications for depression." "Women often become more anxious toward the end of their pregnancy," or "Some depression is common after childbirth," do not address this patient's realistic concerns. The correct answer is: "Did you experience any emotional difficulties after the birth of your other children?"

A woman in the 7th month of pregnancy with her third child tells her physician she is worried that she will experience depression after the child is born. The most important thing for the doctor to say at this time is Select one: a. "Do you want to start taking antidepressant medication now?" b. "Did you experience any emotional difficulties after the birth of your other children?" c. "Most women who worry about depression never experience it." d. "Do not worry, there are many effective medications for depression." e. "Some depression is common after childbirth." f. "Women often become more anxious toward the end of their pregnancy."

Saying "Let's talk about the pros and cons of getting a tattoo will encourage the girl to talk about her motivation for getting the tattoo. The current risk of getting the tattoo or problems with removal of the tattoo in the future prob ably are not as important at this time as why she wants it so badly. Saying "I strongly recommend that you not get the tattoo" or criticizing her by saying "If you know there are risks, why do you want the tattoo will not be effective. Just giving her a brochure also will not be helpful; most likely it will be ignored. The correct answer is: Say "Let's talk about the pros and cons of getting a tattoo"

A15-year-old girl is brought to the doctor by her mother because she is insisting on getting a tattoo. The teen ager states that she knows there is a risk of human immunodeficiency virus (HIV) infection but wants to get the tattoo anyway. What is the doctors best next step in management? Select one: a. Say "Tattoos are permanent and can rarely be completely removed b. Give the teen a brochure describing the health risks of tattoos c. Ask if you know there are risks, why do you want the tattoo? d. Say "I strongly recommend that you not get the tattoo" e. Say "Let's talk about the pros and cons of getting a tattoo"

Because the behavior (hitting the dog) is increased, the scolding that this child received is probably positive reinforcement. Both negative and positive reinforcement increase behavior. The reward or reinforcement for this hitting behavior is most likely to be increased attention from the father. Punishment decreases behavior. The correct answer is: positive reinforcement

Although a father spanks his child when she hits the dog, the child continues to hit the dog. This child's hitting behavior is most likely to be a result of Select one: a. classical conditioning b. negative reinforcement c. sensitization d. extinction e. habituation f. shaping g. punishment h. positive reinforcement

Extinction is the disappearance of a learned behavior when reinforcement is withheld. The father's intervention, which led to improvement in this boys behavior, can best be described as extinction. Initially, the child's bad behavior increased in order to gain a reward (attention, even if it was scolding). When the father stopped reinforcing the child's bad behavior and instead ignored it. the behavior ultimately disappeared (became extinct). In positive reinforcement behavior is increased by a reward; in punishment behavior is decreased by an aversive stimulus. Modeling is a type of observational learning. Shaping involves rewarding closer and closer approximations of the wanted behavior until the correct behavior is achieved. The correct answer is: extinction

Although he is scolded by his father for watching television when he should be doing his homework, a 9-year-old boy increases his television watching. The father then decides to ignore the boy's television-watching behavior. Within a week, the boy has stopped watching television when he should be doing homework. The father's intervention, which led to improvement in the boy's "doing his homework" behavior, can best be described as Select one or more: a. modeling b. punishment c. extinction d. shaping e. positive reinforcement

This 80-year-old fem ale patient is showing signs of Parkinson's disease (eg, a resting tremor, little facial expression, and problems initiating movement). This disorder is associated with abnormalities of the basal ganglia. The correct answer is: basal ganglia

An 80-year-old female patient has a resting tremor of her left hand, little expression in her face, and problems taking a first step when she has been standing still. The area(s) of the brain most likely to be affected in this patient is (are) the Select one: a. right parietal lobe b. left frontal lobe c. amygdala d. basal ganglia e. reticular system f. hippocampus

This child is likely to show reactive attachment disorder after this prolonged separation from his mother. Although the orphanage is well kept, it is unlikely the child has been able to form a stable attachment to another caretaker because of the high number of staff changes. Loud protests occur initially when the mother leaves the child. With her continued absence, this child experiences other serious reactions. These reactions include depression, decreased responsiveness to adults, and deficits in the development of social and motor skills. The correct answer is: Reactive attachment disorder

An American couple would like to adopt a 10-month-old Romanian child. However, they are concerned because the child has been in an orphanage since he was separated from his birth mother 5 months ago. The orphanage is clean and well kept but has a high staff turnover ratio. Which of the following characteristics is the couple most likely to see in the child at this time? Select one: a. Typical development of motor skills b. Increased responsiveness to adults c. Loud crying and protests at the loss of his mother d. Reactive attachment disorder e. Typical development of social skills

Amyloid plaques are seen on brain biopsy of Alzheimer's disease patients. The correct answer is: deposition of beta-amyloid

The postmortem brain biopsy of patient with Alzheimer's disease is most likely to show Select one: a. right frontal hypertrophy b. decreased homovanillic acid (HVA) c. decreased calcium levels d. increased 3-methoxy-4-hydroxyphenylglycol (MHPG) e. deposition of beta-amyloid

The unconditioned stimulus (pizza) produces the unconditioned response (stomach growling in response to pizza). The unconditioned response is reflexive and automatic and does not have to be learned. The unconditioned stimulus (pizza) is the only element here that by itself will elicit a natural GI reflex (stomach growling). The white van is an example of the conditioned stimulus. In this scenario, the conditioned or learned stimulus causes the same response as the unconditioned or unlearned stimulus only after it is paired with pizza (stomach growling in response to pizza). The correct answer is: unconditioned stimulus → pizza, unconditioned response → stomach growling in response to pizza, conditioned stimulus → the white van

For the past year, pizza has been sold from a white van outside a high school. The teenage students complain that they are often embarrassed because their stomachs begin to growl whenever they see any white vehicle, even on weekends. The principal then bans the van from selling pizza near the school and the students' stomachs stop growling at the sight of white vehicles. Match the following learning theory terms with conditioned/unconditioned stimuli and responses in this scenario: unconditioned stimulus Answer 1Choose...stomach growling in response to pizzapizzathe white van unconditioned response Answer 2Choose...stomach growling in response to pizzapizzathe white van conditioned stimulus Answer 3Choose...stomach growling in response to pizzapizzathe white van

Most children are toilet trained by the age of 5 years. However, bed wetting in a 5-year-old who has never been toilet trained and is otherwise developing appropriately is most likely to be a result of physiological immaturity, probably related to genetic factors, for example, the father was also a bed wetter. Emotional stress, sexual abuse, and depression are less likely to be the cause of bed wetting in a child who has never been toilet trained, although they can lead to bed wetting in a previously toilet-trained child. Absence of medical findings indicates that this child is unlikely to have a urinary tract infection. The correct answer is: physiological immaturity

The concerned parents of a 5-year-old child report that the child is still wetting the bed. The child is otherwise developing appropriately for his age and physical examination is unremarkable. The child's father was also a bed wetter until age 8 years. The most common cause of enuresis in a child of this age is Select one: a. physiological immaturity b. sexual abuse c. urinary tract infection d. major depression e. emotional stress

The typical 2-year-old child will see the death of the grandfather as abandonment, while the 4-year-old child will see the death as punishment. It is not until after age 6 yearsthat children begin to understand that death is final and irreversible. The correct answer is: is abandonment: is punishment

The elderly grandfather of two children, one aged 2 years and one aged 4 years, has recently died. The children's grandfather was very involved in the care of the children. When the children are told about the death, the typical perception of their grandfather's death for the 2-year-old and the 4-year-old. respectively, is most likely to be that the death Select one: a. is irreversible; is punishment b. is abandonment: is punishment c. is punishment; is abandonment d. is abandoninent; is irreversible e. is punishment; is irreversible

This woman is showing evidence of a serious postpartum reaction such as major depression, not simply the "baby blues." Because she shows signs of depression, for example, early morning awakening and lack of appetite, the next step in management is to assess her for thoughts of suicide. The child must also be protected. If she is suicidal or likely to harm the child, inpatient treatment maybe indicated. Ultimately, assistance with care of the child maybe helpful, but the first step is to protect the patient and the child. Just setting up another appointment for the following week or prescribing an antidepressant will not protect either. The correct answer is: assess the patient for thoughts of suicide

The husband of a 28-year-old woman, who gave birth to a healthy infant 2 weeks ago. Reports that he found her shaking the infant to stop it from crying. When the doctor questions the woman about the incident, she says "I did not realize it would be so much work." The patient also reports that she wakes up at 5 am every day and cannot fall back asleep and has very little appetite. The next step in management is for the doctor to Select one: a. assess the patient for thoughts of suicide b. prescribe an antidepressant c. set up another appointment for the following week d. advise the father to hire a caregiver to assist the mother in caring for the child e. tell the father that the mother is showing evidence of the 'baby blues'

The most important psychological task of infancy is the development of an intimate attachment to the mother or primary caregiver. Stranger anxiety, which typically appears at about 7 months of age demonstrates that the child has developed this attachment and can distinguish its mother from others. Speech, the ability to think logically, and the development of a conscience are skills that are developed later during childhood. The correct answer is: an intimate attachment to the mother or primary caregiver

The most important psychological task for a child between birth and 15 months is the development of Select one: a. a conscience b. an intimate attachment to the mother or primary caregiver c. speech d. stranger anxiety e. the ability to think logically

The physician should reassure the mother that all children are different and that some crying is normal. The child's appropriate weight gain and negative medical findings indicate that the child is developing typically. Once the mother is reassured, it will not be necessary to recommend a psychotherapist, prescribe an antidepressant, refer her to a pediatrician specializing in 'difficult' infants, or recommending that the father care for the child when it is crying. The correct answer is: reassure her that all children are different and that some crying is normal

The mother of a 1-month-old child, her second, is concerned because the baby cries every day from 6 pm to 7 pm. She tells the doctor that, unlike her first child who was always calm, nothing she does during this hour seems to comfort this baby. Physical examination is unremarkable, and the child has gained 2 pounds since birth. With respect to the mother, the physician should Select one: a. prescribe an antidepressant b. refer her to a pediatrician specializing in 'difficult' infants c. recommend that the father care for the child when it is crying d. recommend that she see a psychotherapist e. reassure her that all children are different and that some crying is normal

The doctor should reassure the mother that sexual practicing behavior with same or opposite sex peers is common and typical. Therefore, it is not necessary to talk to the boy or test his androgen levels. The boy may have a homosexual or a heterosexual orientation in adulthood. Either orientation is typical. The correct answer is: Reassure the mother that this behavior is typical

The mother of a 12-year-old boy comes home early from work and finds her son taking a shower with another 12-year-old boy. The mother is very upset and immediately calls the pediatrician. At this time, the doctor should Select one: a. Ask her to come in with her son as soon as possible b. Ask her to bring her son in and then let the doctor speak to him alone c. Test the boy's androgen levels d. Reassure the mother that this behavior is typical e. Tell the mother that the boy will probably have a homosexual orientation in adulthood

It is typical for a 3-year-old child to have difficulty sitting still for any length of time. By school age, children should be able to sit still and pay attention for longer periods of time. Thus, this is not ADHD. There is also no evidence of delayed development, lack of basic trust, or separation anxiety disorder. The correct answer is: Typical behavior

The mother of a 3-year-old child tells the doctor that, although she instructs the child to sit still at the dinner table, the child cannot seem to do so for more than 10 minutes at a time. She squirms in her seat and gets out of her chair. The child's motor and verbal skills are appropriate for her age. Which of the following best fits this picture? Select one: a. Attention deficit hyperactivity disorder (ADHD) b. Separation anxiety disorder c. Lack of basic trust d. Delayed development e. Typical behavior


Ensembles d'études connexes

Endangered Species: The African elephant

View Set

2.3 Summarize secure application development, deployment, and automation concepts

View Set

Quiz 1 Male/Female Reproductive System/Disorder

View Set